Here’s a collection of questions and answers I’ve given, in a handy sortable table. Here’s a Notion table you can make a copy of.
To get new questions and answers, make sure you subscribe to my Q&A Newsletter. To submit your own question you want me to answer, contact me here.
Date | Topic | Question | Answer | |
8/7/2022 | Extracurriculars - Academic Competitions | I'm a 10th grader that's planning to participate in the National Chemistry Olympiad next year, and I was wondering what chemistry textbooks you would recommend in order to best prepare for the exams. Thank You. | I personally used the book my AP Chemistry course used, Zumdahl. The test may have changed a bit since the nearly 20 years since I took the USNCO, so this might not be the best book now. Whatever you use, the best training resource are real sample tests - these will show you where your knowledge gaps are and where you have to focus your training. | |
8/7/2022 | Coursework - APs | For context, I'm aiming to eventually go to a T20/Ivy school. I personally want to major in STEM, specifically astrophysics. I'm a current sophomore, and I'm filling out my course selections for the next school year. I currently have AP Physics C, AP Chem, AP Lit, AP Span5, and Orchestra. Here's where I'm stuck. I have two hours left, and ideally, I'd fill those slots in with Calc3/Astronomy (both semester-long) and AP Music Theory. However, my parents are quite passionate about AP Statistics and AP HumanGeo. Their reasoning is that two music classes will look bad, APStats is an extra AP, and I need another year of social studies. Let me break this down a bit. They're agreeing to let me take APMT if I drop my orchestra class, but that class is a requirement for a string group I'm in. I'm passionate about that, and I'm not willing to drop it. They think that APMT is unnecessary and a waste of time, especially since I have orchestra already. My dad thinks I should wait to take Calc3 in my senior year, because admissions won't see my APStats score if I take stats in senior year. He also thinks that I won't be able to handle Calc3 (I ask him for help in APCBC, but I have an A in the class), but I know I can if I put in effort. They want me to take APHG (or even humanities) for an extra year of social studies. I already have my 3 credits (currently in APUSH), but since I took freshman history in eighth grade, it wouldn't count as a class 'taken in highschool' (?), so APHG would be my third year (but 4th credit). | From an admissions point of view, you have enough AP courses that this micro-optimization won't really make that much of a difference. Each marginal change isn't going to make a meaningful difference in how an admissions counselor views you as a candidate. For instance, going from 8 to 9 AP classes won't make much of a difference. If you've taken AP Calc BC and done well - and have good SAT Math scores - taking AP Stats is not going to add much to the "this student can handle math" interpretation. On the flip side, if you're good at an instrument and have the achievements to show for it, then AP Music Theory isn't going to add much to the "this student really likes music" interpretation. In this situation, I'd optimize as follows: -take classes that are requirements for what you want to do -avoid classes that don't add much to your application, but will suck up a lot of time and energy - this time is better spent on deepening your extracurricular achievements | |
8/7/2022 | Extracurriculars - Academic Competitions | I'm writing because I'm interested in competing in science competitions like you did in high school. I am interested and motivated in science however I have no idea how to get to these prestigious science competitions. The first thing I want to know is specifically what I should study (books/past papers) . I would like to know what you studied for the US national chemistry olympiad and how I can find good books and websites that will prepare me well for competitions such as these. I'm quite interested in all the three main branches of science so feel free to recommend any science book you find to be good study material for these levels. | There are a LOT of resources out there to prep for every competition nowadays. Google "how to study for the UNSCO", study the top 30 links and the resources they recommend, and you will already be way ahead of average. The information is all out there, most people just don't make good use of it. | |
8/7/2022 | High School Choice | I wanted to ask if you left a stem-focused high school and changed to a public high school the following year, would that look bad if you were applying to engineering schools? | It doesn't look bad if you have a good reason for leaving that's largely out of your control - say, your parents have to move - AND if you can achieve equally in public school, with good standardized test scores and eng-focused extracurriculars. It can look questionable if you left because you were struggling academically at a very demanding school, and if you then don't show your academic skill with good test scores. In other words, going to public high schools is obviously not a downgrade in your chances - if you're a strong student, you can succeed in every environment. | |
8/7/2022 | Extracurriculars - Academic Competitions | I want to participate in the Chemistry Olympiad as a sophomore, but I am taking Chem Honors (students are only allowed to take science AP classes senior year) next year, which I know does not match the level of difficulty that USNCO has. Having attended the study camp yourself, how would I suggest I prepare for it, especially since the classes at my school next year may not be enough to give me the necessary chemistry knowledge? | ----As Above--- | |
8/7/2022 | Extracurriculars - Research | I have also recently started thinking of participating in science fairs such as ISEF, but I do not exactly understand how ISEF participants come up with such complex projects and theories. Also, these projects have to be unique, new, and innovative. Do you have any tips for coming up with ideas? | I sense that the majority of projects are heavily influenced or assigned by the research supervisor, and only a minority of really good ideas come mainly from the student. Doing really original things in science typically requires a lot of background knowledge on what's been doing before and where the exciting directions are, and this is hard to access for most high schoolers since they just learned what a Golgi body was a few years ago. So it's no accident that the children of academic professors tend to do well in these competitions. With that said, it's still possible for you to come up with ideas even if you don't have access to such a mentor right now. I recommend: - read a LOT in your field. Read summaries of recent research in your field (these are called "reviews"), read popular media writeups of recent research. The actual primary papers may be too technical for you to understand, but you can read the abstract and get a gestalt for what they did and why. - think about interesting questions. Your lack of knowledge is a benefit - you can dare to ask questions that seem ridiculous to the expert but have an interesting wrinkle to them. - send those questions to research professors in your field, papers you've read about. Most of them won't respond, but some of them may take the time to and point you to opportunities | |
6/17/2022 | Choosing a College | I applied to all the Ivy League universities this year, along with Stanford, MIT, and other top universities in the US, with a SAT super score of 1550 and my most outstanding extracurricular being a National 3rd prize in an All India Debating competition that I participated in, in 10th grade. However, I didn't get accepted anywhere and now, I don't know what I should do. I've always wanted to study at MIT, Stanford or Harvard, and so, I feel I shouldn't give up and that I should try again next year. But having gotten a year more than the other applicants, and more time each day, since I don't have school anymore, I guess much more would be expected from me. There's also the fact that I only have a few months before the next admissions cycle to achieve something. All the same, once I know what I should do, I am ready to do all that's necessary to be accepted next time. What do you think I should do, Mr. Cheng? Is it really of any use to try applying again? And if it is, what kind of activities are rigorous enough to give me a good chance of getting in, in spite of being a year late? | If you were already rejected, you will need to dramatically increase the strength of your application to have a chance of getting accepted. This often means deepening your achievement in your area of passion, or correcting any major flaws in your application if that is what set you back. More realistically, though, I recommend you move on and attend college at the best place you can. People tend to fixate too much on getting into a place like Harvard as the final goal, when in reality Harvard is merely one of many stepping stones to the final goal: whatever you want your life to be about. Harvard is better in some ways to do that, but not 10x better than the best school you can get into. It is entirely feasible to become successful at a non-Ivy League school - it is entirely up to you. | |
6/17/2022 | Tests - APs | I am currently a HS junior and I have taken 1 ap test in freshman year and i got a 3 and I decided not to take any tests last year because of covid and my mental health. Currently I am planning on taking 5 ap tests, but i don't know if i can get 5s on them. I realize that I should have probably taken them last year but I genuinely could not. I am trying to get into an Ivy League because it has been my dream to for as long as I can remember. Do you think not having AP scores is a deal breaker. I still have to take my tests from this year but would it count against me? | Not taking AP scores is not a deal breaker. Also, when sending scores, you can choose to withhold certain scores from colleges. This isn't perfect - colleges will wonder why you took an AP course and didn't take the test or submit a score, and basically infer that you did poorly. But it won't be a HUGE detractor from your application if other parts are strong. | |
6/17/2022 | Extracurriculars - Spike | However, I read your blog posts about developing a "spike", but my town is so small there aren't many opportunities. There's barely any AP classes at my high school, barely any clubs and organizations. I have straight A's, I have a passion for the flute, Math, and ELA. I also like participating in academic activities/clubs such as BETA club, state Math Team, Spelling Bees, etc. I love helping people, reading, writing, making logical arguments, and just an overall nerd who has a burning passion to learn. But, I still don't know what to do in terms of my spike. I wanted to get a head start on this to give myself more time. I was wondering if you had any ideas for what I could do? Like maybe even over the summer before 9th grade? Also, if I were to participate in the dual enrollment program, could I put that on my college application? Or would it just be useless in terms of contributing to the application. Thanks Allen, love the work you're doing! | Look elsewhere in the Q&A notes for previous notes about this. Colleges will evaluate you on the basis of what opportunities are available to you, so you won't be evaluated on the same footing as a kid in a private school in Manhattan. With that said, there are lots of opportunities available through the Internet. I encourage you to: -read about application stories online to get a range of ideas for what people have done (don't get fixated on application horror stories, these forums can get toxic and neurotic) -identify a few specific interests you have and pursue them to the deepest extent possible in your free time. This might mean learning a lot online and creating something of value and sharing it through the Internet. -find opportunities outside of your town if available. These might be virtual programs, or on-site programs like summer camps or teams, or opportunities in the nearest city. | |
6/17/2022 | Extracurriculars - Spike | I am a homeschooled sophomore who has a 4.0 GPA, I am taking honors (and soon AP) courses, and I have a close relationship with a handful of my teachers. However, in terms of extracurriculars, I am somewhat scattered. I am well aware of the concept of passion projects, but I have no idea where to start or what to focus on for mine. For one, a large portion of my time is spent taking care of and homeschooling my brother who is seven years younger than me (he is currently 9). Because of my “parenting” responsibilities, I have been told I am very mature and have strong leadership skills (this is a factor of my resume I want to show as well, but I am unsure of how to). I am also very invested in cooking/baking, exercising, and organizing/planning. I don’t know how important this is to mention, but from age four to fourteen, my life was heavily embedded in musical theater and acting. Since Covid-19, I have not been participating in that world very much. Academically, I love all portions of school, but specifically Ancient History, Chemistry, Economics, and Math. Career-wise, I know that becoming a doctor would be my best option (I am a creative, but I believe this is a more realistic career choice) and am considering either becoming a gastroenterologist or plastic surgeon. This is a lot of information and I realize that what I love about your articles (they are clear and concise) is not the case with this email! To summarize, I want to know how to make myself stand out in terms of college applications, but I am highly invested in so many realms of academics and extracurriculars, that I don’t even know where to begin. Essentially, I am a right-brained analyst (the best and worst of both worlds), which makes it extremely difficult to find one area of my life that I could build a college resume off of. How would you recommend going about this? | Jeff Bezos's mom told him something to the effect of, "you can be anything in the world, but you cannot be everything in the world." You have a lot of interests, but to achieve something meaningful in any of them you will likely have to focus. Choose two or three of your interests you have the most energy for and pursue them deeper - think about what the best person in the world in that field is doing and how they got to that position, and use that as inspiration for what you can do to make yourself better. | |
6/17/2022 | Extracurriculars - Spike | I am wondering how important it is that your spike aligns with your major of interest? I spend most of my time dancing; I dance at a studio after school and I am on dance company at my school. This year on dance company I am going to be an officer(service captain). I have also taught at my studio a few times but nothing consistent. This is probably what could most be considered my spike because I love it and it’s what I do with my free time. However, I don’t want to get a degree in dance or anything related to that. Is it bad that my spike and my degree interests don’t align? I also don’t have many awards to show with dance; I’ve won competitions at small local venues but I don’t think colleges care too much about that. Do you have any suggestions? I also love writing and I got a 36 on both the reading and english sections of the ACT, but I don’t think I want to major in english or writing either. I’m stressed because I’m going to be applying to colleges in just a few months and I don’t know what I want to go into or what to do. | It's not important for your spike to be aligned with your major of interest or your future career direction. What colleges want to see is evidence of achievement, because past achievement predicts future achievement. In other words, a spelling bee champion is more likely to succeed as a scientist or lawyer or plumber or craftsman than the average kid out there, because the spelling bee champion likely has higher character traits and raw ability. It's also not critical for you to know what you want to do in life yet. While a lot of students do (or think they do), a lot of people also don't. That's the point of college - to explore and figure out what you like and what you want to do. What you _should_ do in your application is give colleges a sense of who you are as a person and what you like. | |
6/17/2022 | Applications - Extracurriculars | I have a 4.0gpa , 2- 3 hours volunteer work - consistently for 2 1/2 years. I hold an associate board seat on one of those nonprofits. I started a small(very) business. The kids one here seem like they are super power kids, not me though compared to my friends I am!! Seriously I have about 450 volunteer work. I am a top golf caddy in my state, several times recognized. In high school I was on track to do my high school team sports, Basektball and Lacrosse. Covid derailed all that. So now fast forward the last 2 years it being my sophomore and junior year instead of sports at my school - which had limited teams sports because of Covid INSTEAD I skied 3-4 times a week. and I have become a high level advanced terrain park free skier and snowboarder. This is the sport that does all the flips, jumps on on the varied rails. It takes a lot of dedication to achieve my level-- I do not compete. Additionally, I live 75-80 miles from the resorts. So this is true dedication to ski all this time and still have a 4.0 . SO MY ?: This is a huge level activity commitment and I would like to include it on my Common App Activites list - what do you think and how (they have an "other " I believe activity) | If it's something you care a lot about and spend a lot of time in, you definitely should add it to your activities list. Colleges will appreciate that you pursue something out of personal interest and not just because you think it looks good on your application. You might also choose to write about one of these activities as part of your college essay, to give more context around why it's important to you. | |
6/17/2022 | Extracurriculars - Research | I'll be a high school junior this upcoming school year. I was introduced to selective STEM summer programs such as RSI, or SSP really late. I'm currently in the summer of my sophomore year, and I discovered these programs only a few weeks ago. Because I was introduced so late, I honestly do not have many things I've been doing to get into the programs like coding or research experience. As of now, I am working on a good PSAT score, and applying to the MIT THINK program (for research experience), but I'm not really sure what else I could be doing starting this late. I would appreciate your feedback on my situation, and some suggestions for what I could be doing currently this summer to have a better shot at these programs. | These research programs tend to accept the students who have the greatest potential in STEM, which can mean a combination of: -high grades and academic ability, especially in AP courses in STEM -research experience -other extracurriculars, like competitions or teams I've given advice on each of these elsewhere in the Q&A archive, so read through them to find students in similar situations. | |
1/22/2022 | Life Choices | My parents have been forcing me to study pre-medicals in order to get into a medical school. But I JUST DON'T HAVE THE SLIGHTEST PASSION FOR IT. I am into humanities and entrepreneurship. How do I convince them to let me pursue my career goals instead of the emotional blackmail over their career choice? Help! | I encourage you to talk less about the "what" and more about the "why." Why do they care so much that you want to be a doctor? Is it because they think it's a stable, decently-paying job so they don't have to worry for your survival? Is it because they think it's prestigious? Do they want to elevate their own status by having you do a prestigious job? Also talk about what you personally want out of life. Do you want to do fulfilling work even if it's not as stable or classically prestigious? How does what you want differ from what they've wanted for their careers and lives? This type of conversation is sometimes hard to have, but it helps you understand each other a lot better than just arguing about whether you should be a doctor or not. When I planned to leave medical school, my parents and I disagreed about the wisdom of that idea until we had a conversation like this. It was really only then that I appreciated how, as immigrants, what they really wanted to achieve was stability and success for their children, and that a journey into business would risk that certainty. I also articulated to them that life is short, I don't want to spend any time working on things I don't enjoy, and that even if I failed I would easily land on my feet with a decently-paying job outside of medicine; I think they came around a bit after that too. | |
1/22/2022 | Extracurriculars | This year I will be a Junior in high school (homeschooled). I've just started SAT Prep (I'm shooting for a 1500-1550 range, and I am relatively close to that goal) and for classes I'm taking Honors Medieval History, Honors Physics, AP Psychology, AP Biology, AP English Language, and Abstract Algebra (I took AP Calculus BC my sophomore year). I am also captain of my high school Mock Trial team (I compete with my zoned high school and we got #5 out of 100+ teams in the state of Florida last year and we are aiming on making it to nationals this year), I tutor math for free at my local middle school and high school, I am the Senior Patrol Leader (president, basically) of my Boy Scout Troop (I am also an Eagle Scout) and I have hundreds of volunteer hours with them, and I volunteer at the labs of my local state college (currently doing research at their microbiology lab). I wish I could make this post more concise, but I have several unconnected questions to ask. 1.) I am thinking about auditing in college level courses in my state (Florida), either at University of Central Florida or University of Florida (the #1 college in the state). Since I will be done with Abstract Algebra (I am taking it on Art of Problem Solving) by the spring semester of my Junior year I will be completely maxed out of the courses I will be able to take for math, so I am running low on options. Is there an unspoken rule or judgement that admissions departments make about students who dual enroll vs audit courses (since audited courses don't have a grade)? 2.) To give a tiny bit of background, both of my siblings are now in the Ivy League. One of them is at Princeton (English major) and the other is at Cornell (Art/Architecture major). My sibling who is at Princeton mostly focused on her writing in high school and won numerous contests, and my other sister at Cornell was a prodigy-level art student who mostly focused on her portfolio. Unlike them, my resume seems slightly "scattered" and unfocused, mainly because of my broad range of interests; I love math and I am taking higher level course work and I am competing in math competitions, but I also love law, medicine and volunteer work as well. Will this hurt me in the admissions process, specifically my lack of "depth" in a specific field? 3.) I'm thinking of applying to research camps over the summer, RSI being one of them. Do these camps actually matter? I know numerous people at prestigious colleges (Harvard, Princeton, Cornell, MIT, etc.) from a variety of connections, and after speaking to all of them the vast majority never did any of these camps such as RSI, PROMYS, MIT PRIMES etc. Is there considerable value to attending these camps? | 1. I don't know for sure, but I imagine audited classes don't add that much to your candidacy. They don't prove that you've learned anything, just that you paid the registration fee. I suggest that you take your interest in math in a more productive direction, such as doing math research, applying math to some type of project, math competitions, etc. 2. Broad interests only 'hurt' to the extent that they take away time that you could spend on deepening your achievement in a particular area of passion. I'd usually recommend scrapping time from things you care less about and spending it on a singular focus you like a lot. But if you don't like anything in particular, then you don't need to do this. I write about this a lot in my guide: https://blog.prepscholar.com/how-to-get-into-harvard-and-the-ivy-league-by-a-harvard-alum 3. None of these are sufficient or necessary to get in - getting into RSI is no guarantee of getting into Harvard, and plenty of people get into Harvard without attending one of these. But if you can get in, from an admissions point of view, they serve as a big stamp of approval from an admissions committee that colleges respect. But even ignoring the admissions value of these programs, they're simply super fun to be a part of. I grew up in a suburb and went to a competitive-but-not-elite-tier high school. When I went to USNCO and RSI, they were the first times I was surrounded by people who were uniformly hyper-competent, interesting to talk to, and from diverse backgrounds. They opened my eyes to a greater world outside of my little suburban bubble. | |
1/22/2022 | Extracurriculars | I am an 11th grader at a high school for Gifted students in Vietnam. I didn't study AP or IB and just studied the normal education system in my country. I don't know if this makes the admissions officer underestimate me. I have a passion for Education, Geography, and Computer Science. I spend time on each one and I love all three of these passions, very passionate. But is it too much? I also do not have many outstanding achievements related to these 3 passions. This makes me feel like I'm not good at anything that stands out. I want to study Business Major at Harvard, I really like this field and I feel that this field and my passion are related. But I don't know how to connect them. I feel like I'm trying to go the more well-rounded path. I took part in organizations related to Education and STEM and found two non-profit organizations about donation and STEM Education. I want to change this. But now, in my country, there are very few opportunities for me to develop it, for example very few internship opportunities for high school students, few Geography related competitions,... I am looking forward to your advice because I feel a bit helpless right now. | If you can't find ready-made opportunities in your area, I'd recommend that you work on your own projects. Could you code a project that educates people on geography somehow? I'm reminded of the game GeoGuessr: https://www.geoguessr.com/ Feel less like you have to join existing organizations or programs, and try to build something that you think would be of value to the world. You don't HAVE to combine all your interests together in one project - you can just focus on a subset of them, if it makes it easier to get started. | |
1/22/2022 | Extracurriculars | I am an international student applying to 6 US colleges this fall. My favorite college is Stanford due to its intellectual vitality, entrepreneurial scene, and great location in the Silicon Valley. I just want your take on how I can hone in my unique spike. I am MASSIVELY interested in blockchain technology. I am interested in changing the world through blockchain. So far, I personally reached out to two universities near me and performed two remote research internships over the summer. For one of them, I wrote a paper and will submit it for awards, and hopefully publish. The second, I created a digital interactive map showing global CBDC development in countries. I am not sure how I could get credit for that since there is no paper. I am currently creating the first ever global blockchain competition. Lastly, I am also creating my own blockchain that will change sports betting - it will be a site/app that people bet on. How could I best develop this profile to optimize my chances of getting into Stanford? | Blockchains exist so people can build useful things with it, so the best thing you can do is to start building useful stuff. If someone in the 90s told you "I have a massive interest in the Internet and I think it's going to change the world," then the natural response would be, "well go and build something useful with it" and would be less about studying it and writing research papers. Caveat: gambling is an iffy subject in the US - it's commonly seen as a vice and a below-average industry in reputation. If you're passionate about it you should still go for it, you may just have to message it in a more generous light. | |
1/22/2022 | Applications | I have applied to many "safety" schools and have started applying to reach/dream schools and I was wondering if it would be a good idea to apply test optional to more competitive schools. I have a GPA (3.41) that is below average due to circumstances my Freshman year and have SAT/ACT (SAT: 1250, ACT: 26) scores that could be better, especially considering that my GPA is already low. I was wondering if it would be best for students with lower GPAs and average test scores to apply test-optional to top-ranked schools/schools in general? | If you can afford the application fee, then it can't hurt to apply to more schools so you don't have any regrets and "what if" questions. But if you don't have competitive grades and extracurriculars, your chances of getting into elite colleges are small. Keep in mind part of the reason colleges have test-optional policies is to get more (often less qualified) students to apply, thus lowering their admissions rates, thus making themselves look more selective, thus raising their ranking and prestige. It's not an accident that Harvard and Stanford's admission rates are now below 5% - a lot of students are doing what you're doing because of test optional. | |
1/22/2022 | General | I read your post on how to get into schools like Harvard, Yale, Princeton and Stanford. I’m 62 retired from the County and a disabled Vet of the United States Marine Corps. But I was really searching on how to raise your kids to get into schools like those. I have a two year old grandson that Im raising with the help of my wife who is an assistant principal. Any tips on how to prep your two year old or any kid for elite colleges looking back on your childhood? | Age 2 is probably too early to start thinking about this! Don't worry that you need to do everything perfectly from birth to get into top colleges, you have plenty of time. My best advice is here: https://blog.prepscholar.com/how-to-get-into-harvard-and-the-ivy-league-by-a-harvard-alum I'd say the best thing you can do is to expose the child to a lot of experiences, help them figure out where they have talent and interest, and then nurture them to deep achievement in those areas. This is the same process I'd use when I have kids, and if they did that well I'd be happy regardless of which college they went to, if they even chose to go to college. | |
1/22/2022 | Extracurriculars | I’m a high school junior currently. I was initially planning on developing my spike towards medicine/dentistry as I thought I had a passion for such; however, I took an internship in the summer at a dental clinic, shadowed my mother, which is a radiologist, and I concluded that I do not have a passion for it. I did some reflecting for the past few weeks to see where I felt most passionate in regards of a culmination of tasks and projects I have done, and I found myself to have enjoyed the digital design course I took in grades 9 and 10. Additionally, I currently take a higher level Business Management IB course and also enjoy it. Further to that, I partook in a few higher level Economics IB course lessons and enjoyed them greatly as well. My issue is that, I don’t know which particular field I have a greater passion for (Web development/comp sci VS business/economics/commerce). I cannot switch to supplementary courses, such as Physics for computer science, or Economics for the latter, and so I’m not sure if that would be a weakness in my application (aka to not pursue courses that are complementary to my now desired path). Additionally, because I need to get working on my spike - as I have less than a year to applications due - I don’t know what extracurriculars to pursue without wasting time, because my interests are in two separate fields. | Good for you for ruling out a career path early! Since you don't have a lot of time, I'd try to focus on the one area you feel you can make the most progress in in the upcoming year. It's not necessary to pursue ALL your interests - for the purposes of college applications, it's just important that you show your _capacity_ for achievement, not that you do it in every interest you can possibly have. Pick the one you're a little more advanced in or interested in, and just drive deep into that for the next year. | |
1/22/2022 | Extracurriculars - Academic Competitions | I'm currently a 9th grader. I have a question about where do you practice/prepare chemistry so that you are ranked as #6 in the US National Chemistry Olympiad in your junior year? Did you just master everything in AP Chemistry class in your sophomore year? Also, is this the main achievement that helped you get into the RSI program? | I haven't looked deeply into this for many years, but my stance - to get into the USNCO study camp, you'll likely need to master AP Chemistry to the point that the AP exam is trivial and you get nearly 100% of the questions right. In addition, you'll need to study topics not on the AP, like basic organic chemistry. The best way to study for this is to take USNCO practice exams. Since I was in high school, there are many more free resources available on the Internet, if you just GOogle it. Re: RSI, what they probably liked most about my application was USNCO and my prior research in computer science; I can't tell which they liked more, but competitions like USNCO are nice in that they're standardized and credentialed so it's easier to compare people on an even playing field, compared to something more variable like research. | |
1/22/2022 | Extracurriculars - Spike | What should I do? Do I have a low, decent, good, or great chance at getting into Harvard? I read your article about the spike in the application, but a lot of my extracurriculars/achievements and academics don’t go together. Summarized Academics: B- in English 9 Honors (freshman) 1 B, 1 B+ in Geometry/Algebra 2 Honors (sophomore/junior) 1 B+ in General Science Honors (freshman). 2 A+ in AP World/Euro (sophomore) 1 A- in AP American History (freshman) Summarized Extracurriculars: Signed to a professional organization at the age of 16 (one of the youngest in the entire world) for a game (League of Legends: Wild Rift (mobile)) by one of the largest publishers in the world. I achieved many national awards, runner up national awards, and have even been to international championships (I will not know if I will get 1st out of 24 teams or top 5. Does this matter?) Passion Project: Researching my school district’s course curriculum, specifically American history K-12, and finding first-person sources that I can add into the curriculum to broaden ethnic studies. Went to the school board (in a conservative area) to fight for the addition of my research in order to teach future generations in hopes of eliminating racism and spread the stories of minorities and their ancestors’ successes and failures while in the United States. Question: Basically, none of these things really go with each other. I can’t focus in humanities or STEM because I have B’s in both of these areas. I want to become a lawyer so I can help underrepresented minorities including the LGBTQ+ community (which I am apart of) get the representation they deserve. I don’t know what I should do in order to get into Harvard with a big spike. I don’t even know if I even have a spike. | Your gaming achievements are the most impressive thing I see so far - to do anything on a professional/international level qualifies as a "spike." Gaming used to be less prestigious a decade ago, but it's now significantly more mainstream. The more popular the game is, the more impressive the achievement is, and League is obviously very popular (I don't know how relatively popular Wild Rift is). If your academics and scores are strong, I think you'd be competitive as is with your gaming achievements - you could write essays about lessons you learned from gaming that give it more gravitas than the layman perceives (the importance of grit, teamwork, your exposure to different cultures through your competitions, etc); get a strong rec letter from your coach, etc. You could also push further in your future career direction, around law and activism. These might include getting directly involved in activism or education in areas you're interested in. Or maybe combine the two - is there a need to create more LGBTQ+ representation or awareness in e-sports? If you focus on pain points that you feel yourself, I'm sure you'll come up with creative ideas. | |
10/10/2021 | Extracurriculars - Spike | For context, I am a rising senior at a top New England boarding school(17F). I have read all your Prep Scholar guides, and specifically have questions regarding my spike. I am an avid STEM student–I am a captain of a FTC robotics team that has ranked top 4 in the world championship for two years in a row (we have qualified for the past 4 years, however the past two world competitions have been cancelled due to covid) We consistently place top in state, and have collected accolades from local to state to world level. To me it seems this would qualify as a spike when supplemented with other work I have done in STEM (including a GS gold award and independent studies) First, is it bad this is a team EC rather than individual? Secondly, my middle school science preparation was lacking, and as such my semester 1 grades in science classes trend lower than in other classes (B, B+, A-). I have worked to address gaps in these classes (all honors and APs) and ended with an A for my second semesters. Does this deteriorate my STEM spike? I am also conflicted with how my passion for humanities fits in with all of this. Humanities (and history specifically) has always come a bit easier to me. I have excelled in my high school curricula in this field over the past three years whereas I faced challenge in some sciences. My spike in this field is my original research. I completed a 25 page research paper for publication in a quarterly academic journal, and my paper earned me an internship this summer at a museum in a local historical town conducting research through their archives (topic still deciding). Although I understand this spike is not world class as my robotics spike is, I think it is relevant. While it might seem like overkill to consider both of these spikes, they are both genuine interests. Whereas your science research passion was a clear spike for you, do my two spikes deteriorate each other and make me effectively a jack of all trades? I'm hoping not as I'm hoping to fit them together and study at their intersection in college. TLDR: My science spike is world class whereas my humanities spike is impressive but my math/science grades are lower than my humanities grades. How do I use both strong suits as benefits? | It's not bad that it's a team EC, but you can strengthen that part of your application by discussing your personal contributions to the team. Leadership positions like being captain also help a lot. If you have good AP scores and final grades in science courses, then that should be enough to compensate for a poor start. It's worth discussing the weaker grades in the place of your application that lets you write special notes. Re: humanities - the big picture is that you should do what you're interested in. Life is too short to forsake what you're passionate about, especially for a short-term goal like college admissions. (Ironically pursuing genuine interests is actually the best way to get into top colleges.) And if you have interest in exploring both in college then you should certainly pursue both now and present as such to colleges. But to talk technically about admissions - the point of a spike is to show impressive capacity for deep achievement. The main reason I recommend against developing too many spikes is that the student ends up diluting time across too many activities and becomes mediocre in all of them. In your case, if you feel you have a spike in robotics/CS already and can't go much deeper by spending more time there, then showing different areas of interest won't hurt you. | |
10/10/2021 | Extracurriculars - Academic Competitions | I was reading your ‘Successful Harvard Application’ on PrepScholar when I came upon the USNCO competition. I had already heard about this from a few friends, and after reading your article I became even more interested in the competition. I will be a Sophomore next year in High School, but was wondering if you had any books that you would recommend studying from for this competition, as well as any other tips/tricks you would be willing to share. | The main way to prep is like any other test prep - get your hands on real exams and high-quality test questions. Do a lot of them. Figure out your weaknesses and drill those to perfection. By the time you will take the real tests, you should have taken literally every official test available to you (there must be at least 15 years of these tests), and you should be scoring high enough to pass the historical qualifying bar. If you don't hit this in practice then you probably won't make it on the real thing. As a rough benchmark, to do well enough on USNCO to get into study camp (top 20 nationwide), getting a 5 on AP Chemistry and 800 on SAT Chemistry Subject Test should be trivial. You will have mastered everything in general chemistry and started dipping your toes into organic chemistry. | |
10/10/2021 | Extracurriculars - Academic Competitions | I am a student who is currently participating in Math Olympiad and Science Olympiad at my STEM school. I know it is practically impossible to qualify for USAMO without doing MathCounts/math olympiad in middle school. But I would like to know what you meant by saying you could have ranked higher if you studied for math olympiad. Can you tell me more? What do you mean? Is it possible to qualify for USAMO without MathCounts? | You certainly don't need to have done any prior program to do well on the AMC/AIME/USAMO. The only thing that matters is how you do today. Now, a lot of talented math students will probably have done middle school math programs, but it's not necessary for you to have done it, and I imagine plenty of USAMO takers didn't do competition math in middle school. What I meant by my comment is that, when I was in high school, I didn't really prep for the AMC/AIME. Back then (in 2003) study questions and resources weren't as freely available online, and my school didn't have a strong history of math competitors, so I didn't really think about training for it. Knowing what I knew now, if I went back and prepped for the AIME the same way I prepped for the USNCO, I probably could have done much better (though I doubt I could have made it into MOP). | |
10/10/2021 | Applications - Early Action/Decision | I am a senior in high school who recently explored your well-known prepscholar post on being well rounded vs. having a spike. As I am in the middle of the college-admissions process, I find myself doubting if I even have a chance at the school I'm planning on applying to restrictive early action (I don't want to lose the chance to perhaps be admitted to tier 2 schools such as Vanderbilt because of applying to a tier 1 school initially). I want to pursue pre-med in college, and am applying to schools as a biology major. I have a well-formed, but not world-class spike in biochemistry research (have won local and state competitions, national finalist in research). I am afraid that my bad score on my AP Calc AB exam last year (3- took the virtual and did not perform as expected) will be a dealbreaker for tier 1 schools as a potential STEM major. I also read in your post that a science focused student should have a 780+ on the math SAT section (I scored a 770). In your opinion, would these two low scores be complete deal-breakers in terms of admissions from the view of a tier 1 school? Would this apply to a tier 2 school as well? | The root of your question seems to be that restrictive early action (REA) significantly improves your chances of admission over regular decision (RD), which is why you're afraid of "losing the chance of being admitted." I don't believe REA changes your chances of admission. If you aren't competitive for regular decision, applying REA doesn't improve your chances by all that much. Think about it from the school's point of view. Say they read an application that ordinarily would get rejected in RD - would they think, "hey, this person's applying REA, we normally wouldn't want them but we should accept them because they're applying REA?" I doubt it. Unlike Early Decision, REA is just a weak indication of interest - they know the student's most likely going to be applying to other schools in RD anyway. "But don't REA students get admitted at higher rates than RD?" At some schools yes - but largely because REA students are overall much more competitive than the average RD student (they've prepped earlier to apply to colleges, they understand the nuances of college applications better). Controlling for the same strength of applicant, I would wager the admission rates aren't too different. So either way it won't make much difference. I would use REA for the school that maximizes both your personal interest and where you feel you have a decent chance of being admitted to. Then apply RD to all the other schools. Then let the chips fall where they may. Now, applying Early Decision might materially change your chances of admission, because this is a strong indication of interest and a certain matriculation for an admission (helps colleges preserve yield rates). But many top schools no longer ED. As for your last questions - I don't think any single bad score is a dealbreaker. A 770 is not all that bad. On the other hand if there is a regular pattern of weaker academic preparation, then your application would be weaker. And lower ranked schools will have lower standards than the top 10 schools. | |
9/26/2021 | Extracurriculars - Spike | I’m a junior at _______ High School in _______Illinois. I’ve been on the Varsity wrestling team since freshman year and I have placed in the conference and was a regional champion my freshman year. My grades hover around from a 3.0 to a 3.3 I have two things that I think make me stand out and may give me a better chance of getting in: 1. In 8th grade I started distributing green Colombian coffee from a warehouse in Houston. When the pandemic hit all my business ended so I decided to launch a specialty roast. I’m getting everthing in place and will be launching this summer. Check out my website: _______ 2. I have analyzing companies and have a deep intense will to understand valuation. I’ve read books by professors like Ben Graham and Bruce Greenwald. I completed a research report last summer that was modeled after a research report that JP Morgan would do. I’ll link it below for your reference. With these qualities in mind, how would I do you think I would look to Ivy League recruiters or other colleges with lower acceptance rates such as Pepperdine, Northwestern or Ucla? | It's great that you've identified your passions, clearly in business. The key here, as with all extracurriculars, is to push deeper into achieving greater, quantifiable results with your work. In business, these days anyone can set up a Shopify site; the key is how much revenue do they generate, and how do they generate it? College want to separate 'fakers' from 'makers' and this is one way to do that. Keep growing revenue and when it gets to be substantive (say above 5-figures/year) it'll be something to pay attention to. You didn't talk much about your grades and test scores but those are still important. 3.3, even including AP courses, is low for a school like Northwestern or UCLA (still quite selective). Look up average SAT scores for those schools to see if you get close. | |
9/26/2021 | Extracurriculars - Spike | I am currently ranked 1st in my class of 700 students, and I am scoring close to 1550 on the SAT right now. I also participate in Science Olympiad, in which I have received several first placements at the state level. In addition, I have reached the national level in several events for TSA competitions (Technology Student Association). Also, this summer I plan to work on COVID-related research projects with 2 different professors, one of which will focus on Nanotechnology with a professor from the University of Maryland, and the other project will focus on cardiovascular surgery-related research with a professor from Georgetown University. Both of these professors have also stated that these works will likely culminate in publications in journals, with the Nanotechnology project possibly being published in the Journal of the American Chemical Society (ACS). However, as I understand from your articles, one major "spike" for college admissions is an Olympiad. I am going to be taking AP Chemistry as a rising junior in high school, and I was interested in pursuing Chemistry Olympiad. Aside from my AP Chemistry coursework as preparation, I plan to diligently work through the Atkins' General Principles of Chemistry book as well as enroll in the AOPS ChemWOOT course. Also, I hope to perform laboratory experiments that my teacher can set up for me throughout the school year. Finally, I hope to apply to several research summer programs for next summer, as I will be a rising high school senior then. The programs that I hope to apply to are RSI, RISE, SSP, HSHSP, and RMP, to name a few. Overall, I am just wondering: 1) Do you think that I have already put together an impressive application portfolio? 2) How would you recommend preparing for the Chemistry Olympiad, and do you think that Study Camp is possible to achieve in one year? (I understand that you accomplished this twice) 3) Would my research be impressive to college admissions officers? 4) With this application, would it be likely that I could be admitted to schools such as Harvard, Princeton, the University of Pennsylvania, and Johns Hopkins? | 1. You have a great foundation as a junior. You've identified your area of competence and interest (science/research) and are taking material steps to deepening your achievement there. 2. It certainly is possible to get into USNCO study camp in one year - I did so my sophomore year, the same year I took Chem Honors in the summer and AP Chem during the year. That said, it is likely much more competitive than it was when I was in high school, given how many more free resources and formal study programs there are. But given your strong academic background, I think you have a good shot. The key as with all test prep is to do a lot of high-quality practice questions and focus on improving your weaknesses. 3. To tell the quality of your research I'd have to know more about it and your role in it (whether you're assisting tangentially or playing a meaningful role), but nominally it sounds good. Things that make research more impressive: publication in high-quality journals; a significant role in doing, even driving the research; entering the research into competitions like ISEF; reputation of your PI; warm letters of recommendation from those professors. 4. See elsewhere for my standard response to chancing. My summary impression is that you have a great foundation, but you still have a lot of work to execute on your ambitions over the next year. | |
9/26/2021 | High School Choice | I am an incoming high school sophomore student, class of '24, and I have a dilemma. My issue lies in where the remainder of my high school career will be spent. I currently attend a public high school in Appalachia where I am at the top of my class with a 4.8 GPA and a 30 on my ACT. At my high school, I am a student who is unchallenged by others for leadership roles and school-based opportunities. As it is, I fully expect to be class president for the remaining years, as well as NHS president, and varsity captain of the soccer team. However, due to the instruction last year, during the pandemic, I feel as though I am not being challenged as much as I would like to be. As a student of academic success and a huge fan of academia, I am now interested in transferring to a boarding academy between my sophomore and junior year for the remaining two years of high school. Allen, do you believe that this is the best path for me? Keep in mind, I believe that I have the necessary activities and academic record to be accepted to very competitive schools. I have always aspired to attend one of these schools and also strive to attend an Ivy League school eventually, as well. Money is not a factor in my decision. | This is the tricky question of "should you be a big fish in a small pond, or risk being a small fish in a big pond?" I've answered similar questions before, so look through the answers. Being a big fish in a big pond is probably best of all, but if you transplant to a new school for junior year, it's unclear exactly how you'll perform - you don't know anyone, you might be overwhelmed, you don't have the same momentum in things like leadership as people who have been there since freshmen. Consider if there's a way to get more out of your current school experience. Could you supplement your high school experience with other activities? Do more advanced coursework in nearby colleges? Find online programs that place you with other talented students? Raise your ACT score from a 30 to a 35? Have you made the absolute most out of every resource that is possibly within your reach? In sum, if you feel the pros outweigh the cons (you'd enjoy a more challenging environment, you acknowledge the risks of possibly not doing as well, but feel the better experience makes up for it), then changing schools sounds better. Otherwise you might try to make the best of your current school situation. | |
9/26/2021 | Tests - APs | Would you recommend submitting any AP scores to Harvard if you have mixtures of 5s, 4s and a couple 3s? Would it be better to submit no scores rather than all the scores, which would not count for anything? What would look better to admissions, no scores at all or just the 5s? What if you only have one 5 so far and have 5 AP classes and test to take in senior year? I don't want to say I will report future scores when I don't know what scores I am going to get at the end of my senior year. | It's a slightly tricky situation. If you've clearly taken AP courses on your transcript, then colleges will probably expect you to have taken AP scores. If you don't submit scores, then the implication is that you didn't do that well, otherwise you'd have submitted good scores. So no scores is kind of a 'soft negative' - they don't know for sure, but they probably know you didn't knock it out of the park. (This is why I think test-optional policies are BS - if you clearly have the capability to take a test but don't submit a score, then there's a negative inference that you scored poorly.) Same goes with just submitting the 5's - the implication is that you didn't get a 5 in any of the other tests, otherwise you would have reported them. My instinct is to send the 5's and 4's and let the colleges come to their own conclusions, since 3 is a mediocre score. Different schools have different policies - some will want you to self-report them all, others don't have a clear policy, so just research all your schools. For schools that insist on self-reporting them all, I would just submit them all. | |
9/26/2021 | Tests - APs | I'm a rising junior and I go to a private school which does not offer any AP classes. However, in my sophomore year, I self-studied for four APs and got 4s on all of them. I know where I went wrong with studying - I started way too late (March or April) and for that limited time I shouldn't have studied for all four. I took AP Calculus AB, AP Physics 1, AP European History, and AP Computer Science A. Next year I plan to self-study for a few exams again, including AP Physics C Mechanics, AP Physics C Electricity and Magnetism, AP Calculus BC, and possibly others (considering APUSH and Spanish), and I believe I can get 5s because I will start studying early enough. Now the question: Do you think I should include my 4s when applying to top colleges or will they make me look bad? I would think it's a good thing that I self-studied to maximize my opportunities and that I scored pretty well, but I know there will be many students who score 5s even on exams they self studied sophomore year. Also, do you think me getting more 5s in my junior year will "neutralize" my 4s and show improvement, or will it look bad no matter what? | In your case, submitting all your 4's sounds right to me. Four AP's taken as a sophomore, in meaningful subjects, is already impressive; that you self-studied is even better. I would note briefly in the application (wherever they let you write custom notes) which exams you self-studied for so they know this. | |
9/26/2021 | Extracurriculars - Sports | If you have a high GPA with numerous AP courses and SAT in the mid 1500s and have soft support from the big Ivy League coaches, is it worth applying to those schools? Do I still have a chance of getting in if my spike is my sport? What are the odds of me getting in? | What do you have to lose? Why wouldn't you apply? My philosophy is that even if the chances are low, it's better to apply and get rejected than to not apply and regret it. If you're good enough at your sport to get recruited, then certainly you have a great shot at getting in. If you're not recruited then you'll have to fall back on the rest of your application, and I don't know how strong that is and what schools you're looking at, but once again I would just try your best and apply. Worst case you get rejected. | |
9/26/2021 | Extracurriculars - Research | I am a rising senior and I was wondering if doing an independent study research paper could help me with my “peak” in my area of interest over the summer ? | It surely can't hurt. The question is the quality of the research paper - a paper that's good enough to rank in ISEF is different from a low-quality paper that doesn't do very much that no one wants to read. | |
9/26/2021 | Extracurriculars - Spike | I have also started a digital design/ content management service which I wouldn’t officially call a business, but I’ve build up a credible portfolio through it and I’m not sure how I would classify that extracurricular activity on the Common App? Thank you and hope to hear back from you soon. | I think "Career Oriented" or "Computer/Technology" can work, whatever fits better. The description of what you actually did is more important than the classification. | |
9/26/2021 | Extracurriculars - Academic Competitions | What exactly did you do to prepare for the USNCO and get into camp? Approximately how many hours per day did you spend on preparations apart from homework and school? I am a rising sophomore who is very interested in Chemistry. I will be taking AP Chemistry next year, but based on some resources I found online, there is a lot more information present in the competition that is not in the course. What were the specific resources you used to prepare? | It's been a long time (nearly 20 years) but I don't remember it being super intense - for my first year, maybe a few hours a week for a dozen weeks, and more intense studying leading up to the exam? But my personal experience is irrelevant - more important is how YOU compare to the USNCO standards to make it to the next round. Some people need less study time and some need more. The main way to prep is like any other test prep - get your hands on real exams and high-quality test questions. Do a lot of them. Figure out your weaknesses and drill those to perfection. By the time you will take the real tests, you should have taken literally every official test available to you (there must be at least 15 years of these tests), and you should be scoring high enough to pass the historical qualifying bar. If you don't hit this in practice then you probably won't make it on the real thing. | |
5/24/2021 | Extracurriculars - Spike | I am a sophomore at a high school in Charlotte, and find myself between the categories of well-rounded and spiked, and am not sure how to hone in on one to be more spiked. As of right now my interests and activities are only related to music, writing, and research. I don't have any other things (nothing like debate, mock trial, etc.) that don't relate to those three themes, and am not sure if that is an okay thing or not... Just a quick overview of what my activities are if you don't mind... Music-related: I play the violin (concertmistress of school chamber orch, all-state/national in the first violin sec. but not a concertmistress), started a platform to allow any-level artists to have weekly recitals on Zoom for free, I train in Carnatic vocal (performed in few places in the country and India, but haven't tried for any competitions) Research + Music Related: To connect to music, and an interest I have always had, is that I have done worked with professors on the study of music therapy, and co-authored a literature review with peers that we are planning on publishing mid-year (haven't tried ISEF or national level science fairs at all yet). Writing: I love reading and writing about and have submitted to a few normal and STEM writing competitions with few prizes (haven't tried for anything national level yet as I am still developing). Random, but something I am really passionate about: Started a non-profit called AlumniAnswers, where we interview alum from esteemed colleges on their experience in high school and college (https://alumnianswers.wixsite.com/advice-1) I see these problems in bold as possible ways to fix my problem, but would that mean I should give up this three-themed trend I have going on, and only stick to one? I know you mentioned in your blog post that being spiked is the best way to go, but I am afraid as a sophomore, I won't have enough time (especially since junior and senior years are heavy on workload) to become a national-level in anything yet... Lastly, this is mainly what I have going, and don't participate in school except for orchestra and officer position in Tri-M. Is that alright, to be doing things more outside of school? | I don't think you necessarily have to stick to just one interest, but just note that if you spread your time thin it'll be harder to get deeper achievement on any one activity. I think your combination of research+music is pretty interesting and ties multiple interests together. Try to push this further into helping with original research (a literature review is OK but not nearly as impressive as original research). I personally think this is probably worth trading some violin practice time for - it's very hard to become world-class in an instrument since it's so competitive, but it's somewhat easier to stand out in doing unique research. | |
5/24/2021 | Tests - SAT/ACT | Will Harvard and other Ivys care about my high school score (IB - 33/45, which is bad :D) if I score 1550+ on SATs? (I’m 23 rn, and I didn’t care about studying when I was 18). If yes, how should I address it, if not to say the truth? Would it make sense to retake the IB exam, so I can get a proper score? (context - I’m 24y/o now) | The SAT score should override the IB score, so I wouldn't worry about that. Do note though that as a 24 year old, they will probably judge you a little differently than high school students - you get smarter with age so an SAT score doesn't mean as much. Make sure you make clear in your application why college is important for you to attend now, and what role it's going to play in your future. | |
5/24/2021 | GPA | I'm currently a sophomore, and due to Covid, I've spent 3 out of 4 semesters in online school. My GPA and academics aren't the best, but would usually be ranked top 5% (top 20) looking at past years. However, with online school, I know several friend groups who have extremely inflated GPAs due to cheating or other unfair test-taking strategies (our class averages for online school are completely dependent on tests). Though I will be returning to in-person school next year, even if I prove to colleges that in-person learning suits me more than online learning, my GPA will never rank higher than those who cheated this entire year. (Just a side note, but my school is notorious for cheating and lazy teachers who don't care to report them) - Should I take easier APs to try and boost my GPA next year (junior year) or stick with the rigorous courses I would usually take (probably low As unweighted)? - I'm quite confident in extracurriculars and leadership since that has been my main focus the past two years, but now I am beginning to worry that my academics won't even make the cut. My main concern is: would having this lower class rank (maybe now top 10% or even 13%) affect my chances of getting into a top-tier or ivy league college? | Sorry to hear about that. I also want to say that it's possible they had high grades without cheating, but I'll take your question at face value. First consider whether it's worthwhile to report cheating, if you have evidence. I wouldn't avoid the courses you want to take just for the sake of boosting your GPA. You might do better than you think, and I don't think this type of gamesmanship around getting into college is worth sacrificing things you're interested in. In terms of class rank - it depends on how many students typically get admitted to schools you're aiming for. If the top 10% of each class tends to get into those caliber schools, then you'll probably be fine. Past a certain point, a better GPA and test scores don't really matter - it then becomes about standing out in other ways. | |
5/24/2021 | Extracurriculars - Spike | I am a freshman student attending high school in New Jersey. I have been able to keep up great grades with an A+ average in only Honors and AP classes. Despite this success, I am still worried about college admissions in a few years. I do not really know what my spike is. I love sports like running and biking, and I run for my school, but I'm not an excellent athlete. I am passionate about science and engineering but I am not sure how I can show this as a spike. My math and science grades are my highest, but I still score very well in my other classes like AP World History. Due to the pandemic, I was not able to do any clubs this year. I am fluent in a few coding languages but I have learned them myself and do not have a course or a program to show it. I also have not competed in any national competition. On top of this, I cannot take a math or science AP course until 11th grade. I don't want to quit sports, but I want to dive further into engineering and science because it is my passion, and can help me stand out in admissions. What do you think I should do? | This question is too general to be able to counsel through email without finding a lot more about your interests, how advanced you are in each area, your rate of growth, etc. This is where admissions consultants can help - I set up a department at PrepScholar just to service this: https://admissions.prepscholar.com/ It sounds like you have a direction to look in already - engineering and science. You can develop a spike in here through any number of ways, such as competitions, building things, or doing research. It's important to pursue your area of interest and not worry too much about what looks good - if you care about it and pursue it deeply, it'll be impressive Try to look at the world leaders in your area of interest, then try to do a fraction of what they do. You might also find profiles of college applicants you respect for ideas - check places like collegeconfidential or relevant subreddits (but try not to get sucked in - these places are often pretty neurotic). | |
5/24/2021 | Extracurriculars - Spike | Hello Allen. I am a sixteen year old student living in Pakistan and am interested in the sciences field. I am currently exploring the engineering field, in particular biomedical engineering. I have read some of your articles on prep scholar, and I had a few questions. I understand that as a student in the US, you had access to many research nd olympiad opportunities that I unfortunately do not have. I am looking into doing a research paper with someone who has a PhD in Chemistry, and also teaching myself coding via codeacademy.com . I would like to know if you have any suggestion as to how to build myself up into a word class student and improve my application for MIT, Harvard etc. Also I genuinely enjoy writing however it does not really align with my career goals, so is that something worth investing in for my application or not? I would really appreciate any guidance you can give me. | The principles of building the spike are the same, regardless of where you are in the world. You'll obviously face some resource disadvantages compared to the US or EU, but colleges understand this and want to see that you made the most of the opportunities you had access to. You might not have access to the exact same competitions as in the US, but I think Pakistan is involved in the major international olympiads and I'm sure there are plenty of national science/eng competitions, so you have some avenues for competitions. Likewise, as an engineer, you can build useful things - for the wider world, or for a local audience. In terms of writing, I'd pursue it as a hobby if you enjoy it. There's no reason to cut things out you enjoy just for the sake of college applications. If it develops further then you might spend more time on it. | |
4/17/2021 | GPA | you have mentioned that class rank is an important factor in college admissions decisions, but my school does not rank students in its classes (I go to a Catholic all-boys school, to clarify). Does lacking a class rank hurt my chances of admission to a top school (Ivy League, Stanford, etc)? I'm a very strong student, having taken 9 AP classes in my sophomore and junior year and earned As and A+s in all of them, but I am not sure if lacking a class rank will help or hurt me in the college admissions process. | Not having a rank isn't going to hurt you, as long as you have other standardized ways to compare yourself with the rest of the application pool, like SAT/ACT/AP scores. It also helps if your school is well-known to the schools you want to apply to, since admissions readers will know how competitive your school is relative to other schools, and thus place your grades into context | |
4/17/2021 | Tests - SAT/ACT | I currently work as a tutor at a Title One school. I am sure you know what Title One schools entails and the status of resources those schools receive.. However, since I work in Orlando, FL it may vary the information or severity of the information you may know about our school systems here. But if you don't, then Title One schools are schools that have students in the lower 25 percentile and I have students who have a myriad of scores. Some receive less than 300 on their SAT and others are 20 pts away from reaching the state standard score of 430. Do you have any advice in addressing the issue of: -How to help improve the scores of the students who all have different learning methods? -How I can best assist them as a first-time tutor? -How to attain the best resources for them without them having to pay out of pocket? | This is a tough problem and admittedly I don't have a lot of experience working with students who are in the bottom 25th percentile. My first intuition is that motivation may be an area to address where it wouldn't be with elite students. If the students you work with have little interest in going to college and don't understand the point of taking tests, then it's probably hard to help them learn no matter how good a teacher you are. Next, I think the low-hanging fruit is basic test-taking strategy. For example: -Make sure students know to answer every question, even randomly. There is no penalty for guessing and they can get free points this way. -At a 400 level, students probably won't get the most difficult half of questions right. It makes more sense for them to focus on the easier half of questions and spend more time on them, than to allocate their time across all questions The best approach is probably for you to learn how to succeed on the test yourself, and then to teach them the methods that are most relevant to them. | |
4/17/2021 | Tests - SAT/ACT | Do you have any info on SAT scores on the students who are categorized as "Americans Abroad"? Any info is appreciated. | I do not, but if you're trying to see how your scores compare, I would assume the percentiles are about the same as for domestic Americans. If I had to guess, Americans abroad would probably be slightly more competitive, since parents who are abroad probably on average have more resources or are more educated than the median American family, but this is an uninformed guess. | |
4/17/2021 | Applications - Chances | I would like to ask if fact that nobody from my country is attending my dream university can give me higher chances to get admitted . When I think about it it can be a good name for them as they have students from more countries. | If your country doesn't typically send many students a year, then your nationality can look a little special. However you will still need to be a really competitive applicant in your own right; between a unique nationality student and a domestic student who is significantly stronger, the school would probably take the latter. Also keep in mind that international students tend to have lower acceptance rates in general, because there are fewer spots open for them. | |
4/17/2021 | Extracurriculars | Do you have any advice for RSI applicants who are selected as Alternates? Is there anything they can do to increase their chances of being moved to the Participant list like sending an email of continued interest? | It's probably similar to letters of interest for colleges - discuss your interest in the program, why you would be a good fit, and how you would definitely attend if selected. Don't expect it to work magic, but it can't hurt. | |
4/17/2021 | Applications - Essays | My question is can you be well rounded but all the activities you do should explain that they relate to each other in the essays and what do you mean by a unifying theme and how do I develop one? And the activities in passionate are entrepreneurship and business and social studies how can these all tie as a ' unifying theme' in my application | A coherent theme is one that ties together your interests and better gives an idea of how you will use your diverse talents for a single focused purpose. It's not strictly necessary, but it helps make you look less scattered and more than the sum of your parts. For instance, someone with a background in computer programming, teaching, and public policy could unite this together with a goal to make computer programming a standard part of the curriculum in elementary schools. | |
4/17/2021 | Tests - SAT/ACT | I took the PSAT in October 2020 and received a 1340 (98th percentile) and the SAT in March 2021, first time, scoring a 1460 (99th percentile). I am a junior in high school and am scheduled to take the ACT on Tuesday, March 30th. I am a PrepScholar client and my primary focus is the SAT, which I will take again in June. I scored 690 on the English section (14 incorrect: 9 reading & 5 writing and language) and 770 on the Math (2 incorrect), so I am aiming to better my English score. Should I still take the ACT? | Generally people score the same on the SAT and ACT, percentile wise, so there's no strong need to take the ACT if you already have a SAT score you're happy with. But if you can afford it and you don't mind taking it, it doesn't hurt - sometimes you might have a marginally higher score on one test than the other. The more important question is whether you have a score that is competitive for the schools you want to get into. If not, then you should focus on studying and mastering one test first. | |
4/17/2021 | Extracurriculars - Spike | I'm a high school junior, and I was thinking of projects I could start. I primarily have a science/engineering background and I had two ideas: 1. Create a COVID-19 video game with a poignant story that would be free for everyone to play 2. Start a nonprofit creating websites for small businesses in my local area I think I would enjoy doing both of these projects, but I think I only have time for one. Lately, I've been thinking about college applications (I'm aiming for T20's) a lot so I was wondering which one would be better for colleges. Just to be clear: college isn't why I am doing these projects, but its a secondary aspect that I want to consider since I will be putting in considerable time and effort. In your opinion, which project would help me stand out and make me a stronger candidate? I understand that you may not know that much about me, so what is your subjective opinion? | The more impressive project would probably be the one that you could achieve more impact in. If you build a video game that gets a million users and press, then that will probalby be more impressive than a nonprofit that makes 5 websites. All else equal, my hunch is that the first (video game) would probably be more interesting to you to build and show your creativity/initiative better. | |
4/17/2021 | College Rejections | So my daughter is senior class president and president of so many clubs. She has a 3.75 unweighted GPA (which I realize is on the low side) was unable to take the SATS or ACT. Well, she is a world-class leader. She's a driven leader and passionate about connecting the divide of educational opportunities between the haves and the have nots that live in our city and go to her school. She is passionate about leadership and making a difference within her school. Apparently, she wasn't world-class enough for any of the highly competitive top schools. State schools rejected her and Georgetown and four ivies (Cornell, Yale, Princeton and Columbia) that she applied to. I am so frustrated as a mother because she has an incredible resume. She has been interning on political campaigns since she was 15 and our city mayor personally called her and offered her a job in his campaign office when he ran for congress. She was promoted to a position that college grads were jealous of. Anyhow, can she appeal a decision from an ivy league? She is nominated for a very prestigious award that recognizes students who have excelled academically and have made a difference on campus and who are the embodiment of what a "stallion" is (our mascot). I was wondering if she won that award and she made an appeal if they might change their minds about her. She has literally dedicated her life to leadership. She is a natural. People and teachers and leaders within our community reach out to her quite often. I don't know where she went wrong. She applied to many state schools that also rejected her..oh, one waitlisted her. I do not get it. Any advice? May we appeal schools like Georgetown and Princeton? | I'm sorry about how disappointing the results are. Because of Covid, schools went test-optional, which made it easier to apply. this then dramatically increased the application count and decreased the acceptance rate. As for your daughter's accomplishments, they certainly are impressive in a vacuum and in your city, and certainly to you as a parent. The difficult context to recognize here is that there are unfortunately many thousands of students at your daughter's application caliber. Keep in mind there are 30,000 high schools in the United States; this means there are 30,000 valedictorians, 30,000 class presidents, tens of thousands of people with perfect GPAs and 5's on APs, tens of thousands of students in the top 1 percentile of test scores. In other words, if all high school valedictorians applied to any Ivy League school, the vast majority are going to be rejected. I say this not to be cruel but to help set realistic expectations that take into account a global context. This is true of any highly competitive process - there are thousands of Division 1 basketball players; only a tiny fraction of them will be drafted into the NBA, and only a tiny fraction of those will be star players. To answer your question - it does not hurt to write letters of interest to schools you want to attend. I wouldn't expect it to change the decision because schools already have long waitlists, but it can't hurt. You can talk specifically about her interest in the school and how you would 100% attend if accepted. Finally, keep this in perspective - the college you go to is just one stepping stone in life, not a linchpin. I know people who went to state schools who are far more successful than my classmates at Harvard. Students who keep working hard despite setbacks will achieve far more than students who peak early and flame out. | |
4/17/2021 | Choosing a College | I was just accepted at Johns Hopkins University (their BME program, which is notoriously difficult to get into) and the University of Pennsylvania. I wanted to ask you if you had any relevant advice on where I should choose to go, as it is a very difficult decision. While JHU is not an Ivy, it is the best in biomedical engineering and Medicine. Does it really matter whether I go to an Ivy or not? | Those schools are really neck-and-neck in terms of reputation so you can't go wrong. There is no particular magic about Ivy League schools. Stanford and MIT are not Ivy League but probably have a better reputation than Cornell and Dartmouth, which are. Keep in mind that you might change your mind about your career. If you go to Hopkins ONLY because of BME, and you decide to leave BME and you hate being at Hopkins, then you might regret your decision. So I'd probably weigh most strongly which school you'd enjoy more overall, and which has the opportunities that you'd most take advantage of. Another way to think about it - think about which school you'd most likely regret NOT going to 10 years from now. | |
3/27/2021 | Extracurriculars | I have three big interests. I am in BTI, so I am interested in science, which is where I hope to find my career. I also am working on changing my school and, hopefully, the district's curriculum regarding Indian studies to be more inclusive and well-taught since it is severely lacking. I also spend a large amount of time outside of school working on my singing. In your articles, you often talk about wishing you didn't spend so much time on violin because you knew it wouldn't go anywhere. My dilemma is that I am unsure whether my singing is good enough to go somewhere or not. I know I won't be pursuing music, though, but I still do high-profile competitions when I can. In your opinion, is it a waste of my time? As you can tell, I am split in many directions and fail to recognize my true path. Even though I have highlighted biotechnology as my future career path, I find that I am spending a lot of time on issues and subjects I deeply care about but would not consider career-wise. How can I find my path? How did you know yours? | There's no accurate way to tell a priori if you'll end up world-class in something or not. You'll have to look at the signals you're getting from the world and adjust. Signals include how you're doing in competitions, how your mentors compare you to other people they know, and your personal rate of improvement. Another way to rule out activities - are you enjoying your time in the activity? If not, then you should probably stop. For finding your true path - 16 year olds shouldn't expect to find their life's direction at their age. The most important thing to do for the next 5-10 years is to try a lot of things, reflect on what you liked or disliked, and iterate again to try more things. I wrote up my personal career path here: https://www.allencheng.com/previous-work/#Why_I_Left_Medicine_for_Business | |
3/27/2021 | Extracurriculars | I'd like to get into top schools including Harvard, Columbia, and Princeton. I want to pursue global public health. I currently have a 1530 on the SAT, a 33 on the ACT, have scored 4s and 5s on all of my APs. I am the founder and president of an Amnesty International chapter at my school, historian for NHS, president of my debate team, and a starting player on my varsity water polo team. In my free time I do a lot of water activities including sailing, scuba diving, and surfing. I have played piano for 12 years and I am an avid artist. Currently, my "spike" seems to be in art. I am my schools "artist" meaning everyone knows that I do art, I painted murals in my school for my IB Personal Project, and I am a self-employed mural artist for public and private spaces. I compete in national art competitions and do pretty well and have gotten recognized on many occasions for my art. However, art is not what I want to pursue––I simply want it to be a hobby. I started Amnesty at my school because it is what I am truly passionate about, but I am afraid it is not a big enough spike to get me into a top tier school. Can you have a spike unrelated to what you want to pursue? And, do you have any recommendations on how I can spike my interest in global public health? | Yes, you can develop a spike outside of your long-term career interests. Colleges care more that you have the capacity for achievement, knowing that you'll probably change your mind about your career choice in college. If you center your application around art, you _will_ have to answer questions about why you're leaving art. You're right that starting a chapter of a club is generally not standout enough to be a spike. Keep in mind there are 30,000+ high schools, and each of them has dozens of clubs all started and led by club presidents. I don't mean to dismiss your accomplishments, just helping you calibrate what a world-class spike looks like. As for global public health, think about what the best people in the world in your field are doing, and try to do a tiny version of that. | |
3/27/2021 | Extracurriculars | I am currently a class 10 student in India. I come from an army background and don't have many opportunities to take as I am constantly on the move, moving to a different school and state every year. As most army background kids in India, I am financially disadvantaged and financial aid will be the only reason I'll be able to attend college. I aim to pursue economics as a major. A few of my accolades include - National Essay Writing Competition runner up ( 2nd out of 22,000 participating students), Starting my own nationally backed NGO for underprivileged girls and women, Writing and publishing my own poetry book and the school leadership council head/president. I wanted to know if these activities help me establish a good spike and if not what are the other activities I can do. Being on the move is really a disadvantage and will that impact my college application? | You have a good foundation and are clearly ambitious. Getting national-level recognition for your activities is a great start to building a spike. Going forward, I would try to tie your work together into a unifying theme, if possible. You're good at writing, you're a leader, you care about social impact. What do you want to achieve in life (if you know), and how do your skills help you here? Then, given that goal, what could you be doing today that prepares you for your life ambitions? This kind of singular vision isn't necessary, but it helps the admissions reader take your experiences and turn it into a coherent narrative. | |
3/27/2021 | Extracurriculars | I am at scout for 8 years and i am thinking of teaching braille method for blind people. Is this a good thing to write in my application? | Anything that showcases your personal interests, character traits, and capacity for achievement is valued by admissions officers. On the other hand, I wouldn't do something just because you think it'll look good (it'll be painful to work on, and you might as well work on something that's impressive as well as something you enjoy). | |
3/27/2021 | Choosing a College | What were your considerations on choosing between all the ivy league schools you got into (specifically Harvard, Stanford, Yale, & Columbia)? Have you changed your mind since then? | Everyone's decisions are different, but here was my thinking process narrowing down the top 10 schools (I will sound obnoxious here, sorry in advance): -I grew up in Southern California in what felt like a social bubble (sheltered suburbia), and I wanted to get far away to establish my independence and see the world. This meant Caltech was definitely out, and Stanford was also kind of meh. Luckily most other schools were on the opposite site of the country. -I visited Yale summer after junior year and didn't like the feel - Gothic architecture and cold. I know this is kind of dumb but it didn't impress me in any particular way. -I got into Harvard early action, and I would attend Harvard over Columbia, U Chicago, or Penn, so I don't think I even applied to those three. -ultimately for me it was between Harvard, Princeton, and MIT. I loved MIT for that rebellious culture, but I felt a technical school would be limiting in case I changed my mind about what I wanted to do (and some MIT people get a little out-there bizarre). -between Harvard and Princeton was a tough choice. Princeton is really focused on the undergrad experience (it doesn't have a law, medical, or business school), and it seemed like a really warm social community (based on the residential college system). -but Harvard seemed like it'd lead to better opportunities: --it had the world's best graduate schools and research institutions nearby, which meant open opportunities for any interest --it was in Boston, rather than rural New Jersey. I wanted to live in a city environment for college. --I could still access MIT resources while at Harvard --at the time, I wanted to get into Harvard Medical School, and this seemed like a more direct route there Ultimately, I asked myself which school I'd regret not going to. If I went to Princeton, I could see myself regretting not going to Harvard. But if I went to Harvard, I couldn't see myself regretting choosing it (it's hard to go wrong with Harvard). That sealed it. Again, I know this sounds like a spoiled kid choosing among riches, but hopefully everyone can take away some general lessons: -the analytical approach: choose whatever school maximizes the things you're looking for -the intuitive approach : think about what place you'd most regret not going to. | |
3/27/2021 | Extracurriculars | Due to the outbreak and everything else that has happened in the recent year and my age as a sophomore, I've been unable to secure opportunities to participate in the world of research. How can I distinguish myself (beyond the perfect test scores/ National Merit)? How should I make use of my summer? I'm located in St. Louis, Missouri, and am applying to the Missouri Scholars Academy. | Lots of students are in your situation, where their lives were upended by covid. Colleges don't expect you to have as strong of an application as if covid didn't happen. They do however care about how you make the most of the situation and the opportunities available to you. Can you demonstrate your interests in ways other than what you had planned? For instance, is there any way to take your interests online, or to do research that doesn't require you to be in the lab? | |
3/27/2021 | Extracurriculars | I know I wanted my spike to be in the medical field specifically research. I have reached out to multiple different doctors doing research in my area and they all say that they cannot allow more people due to Covid restrictions. What should I do? | It's a tough situation, but thankfully if it's tough for you it's tough for every other high school student in the world. A few suggestions: -broaden your search to people further away. All you need is one to work out. -think about what else you can do to demonstrate your interest in this field, other than to do medical research onsite. Could you do more computational research? Create educational content around medicine? Read research papers and write analyses of them? When your options are limited, you'll have to get creative, and you might find new ideas you hadn't thought of. Any such idea executed well will be as impressive, if not more, than doing actual research. | |
3/27/2021 | Applications - Extra Communication | I am an applicant to a couple of colleges: Yale and Princeton this admissions cycle. Last cycle, I was admitted to another fellow Ivy League college, but Yale and Princeton have always been my dream schools. I am contacting you because I am wondering whether or not I should let Princeton and Yale know on my application that I was accepted to another Ivy League. Will that make a positive or negative impact on my application? | By admitted last cycle, do you mean last year? (2019-2020). If so, this information isn't all that relevant to colleges today, because you're obviously not happy with that school and are applying in a new cycle. But I'll answer this for the situation most students will be in - they got into one good school; do they tell their other schools they got in? On one hand, this theoretically _could_help with some social signaling - you were good enough to get into, say, Columbia, so at the least you're at this baseline. On the other hand, if you tell them this info without any meaningful purpose, you'll look like you're scheming and the reader might resent that gamesmanship. My general guidance is to say it only if you have a meaningful reason. Something like, "I have a standing offer at ___, but I would 100% attend Princeton if admitted. You are my top choice." This will never make or break an admissions decision but it can be a very minor factor if you're on the fence. (And please note, you should only send one school this message). Ultimately it's not going to significantly change your application chances, so don't think too hard about it. | |
3/27/2021 | Summer Programs | I am considering applying to summer programs from universities like Columbia, Harvard, etc. I am concerned about the workload with these programs and how much of my summer they will take up. Moreover, I am unsure of the value of these types of summer programs and how much weight they hold on a college application. I know that summer is a really valuable time, so I am wondering what advice you have around the best way to spend your summer, if summer school is really worth it, and what might be some other alternatives(in US and Canada). As well, does it matter if I do summer school in person(if conditions are favourable) vs. online? I am also interested in pursuing something in the field of medicine in university. I want to enhance my resume in this field, and I was wondering if you could give some suggestions of specific opportunities(related to science and medicine) that stand out on a resume. | To put it bluntly, the easier something is to get into, the less valuable it is as an achievement. I don't know about the acceptance rate of these summer programs, but if it's above 50%, practically anyone can get in, so it doesn't do much for your application. It might still be enjoyable, but don't expect it to increase your chances of getting into that school. (Typically these summer programs are revene sources for the colleges and not meant to be distinguishing achievements). Typically the better way to spend your summer time is to focus on an area where you have deep personal interest and make progress there. If that's science research, do science research; if it's nonprofit work, start your own or join one and achieve a lot. Summer school isn't tailored for your interests this way. | |
3/27/2021 | Mental Health | I have Aspergers and ADHD, and it's almost impossible to achieve this when you struggle with daily tasks, such as learning disabilities. Other than taking various supplements and medication, what would you recommend as the best course of action for someone who has a disability? It's easy to build a tolerance to medication, and it just doesn't work the same way from day to day. The same could be said supplements such as various vitamins and fish oil. | As I've said previously to someone with ADHD, I'm definitely not an ADHD expert nor a doctor, so I don't feel qualified to answer this specifically, but I do have three general pointers: 1) Mental disorders are super personal; the same treatment to two different people can have very different outcomes. It can also take a long time to find the best balanced treatment. Find a psychiatrist you like and trust and work together over the long-term. 2) I'd suggest finding other people who struggle with the same things. I know it's hard to talk about this stuff online but there are more people than you think in your situation, and everyone's hesitant to find each other (I get multiple questions about ADHD). You might post on a place like Reddit, like in the ADHD subreddit (https://www.reddit.com/r/ADHD/) or Asperger's (https://www.reddit.com/r/aspergers/). At the very least if you don't want to post, search for related questions to see hwo people deal with it. 3) Colleges are understanding about mental health, and they know that mental disorders are out of people's control. They care more that you made the most of your situation, whatever that was, and showed the character traits that will let you succeed over time. They won't expect you to have as strong of an application as someone without ADHD. Hopefully this takes some pressure off. | |
3/27/2021 | High School Choice | I was applying to 9th grade and now that schools have made their decisions I have four schools to choose from: Cranbrook School, Lake Forest Academy, Western Reserve Academy and Northfield Mount Hermon School. I am interested in working with you and PrepScholar when I will be studying in a school to gradually prepare for college, but for now I have to choose a school that will: help me pursue STEM at a good level; be good overall. | I don't know those schools well so I can't advise on which one is best. Generally, choosing a high school is probably similar to choosing a college: -how well it matches your preferences for a learning environment - location, student body size -the ability of the school to prepare you for your future goals - certain activities you care about, their curriculum -how many students do they send to the caliber of colleges you want to attend All your choices sound like elite private schools so you would probably do fine even if you picked randomly, so see if there are other factors that help you decide. | |
2/7/2021 | Coursework - APs | I am a junior. My school only offers two AP classes, but it offers many courses from the local community college. This college offers many classes like Pre-Calc (that I'm currently taking), Psychology (I'm taking this as well), Spanish, Sociology, English, and Statistics. The Pre-Calc class is good. However, the Psychology course is terrible and is barely teaching me anything. I don't know whether I should take this as a sign and quit doing the courses offered by the college, or power through them, since they're my only chance for college credit besides APUSH and AP Chemistry. Would it hurt my chances in college admissions to not take these courses? My school doesn't offer anything else, and I don't know if I have the willpower to self-study for an AP course. I'd actually like to learn something about the subjects I'm taking, and that might be more plausible at a college. I don't even know if colleges I'm looking at will accept these credits. | If you can change classes, then change out of Psych to another class you like more. If you can't change, but the class is trivially easy, then just treat it like you didn't have the class at all. Spend that time on extra studying for other courses/tests, or on extracurriculars. | |
2/7/2021 | Financial Aid | On to the topic of finance and my prospects. I don't know if my transcript is strong enough for colleges to give me a full ride scholarship. I have a 4.0 (unweighted) GPA and am ranked 4th in my class of 150, but my classes were relatively easy. I don't know if colleges will see that as me slacking on my course load. When I chose my classes, I was unaware that I could double up on math or science. I don't know how that could affect my prospects. If I don't know what to do for college, what should I do after my senior year? After I graduate, my parents have made it clear that I should either go to the local community college and live with them or go out on my own. What are some possible ways to pay for college, even if most schools wouldn't consider me for financial aid? I only want to join the military as a last resort, and I don't know many scholarships that are purely merit-based. (I'm not a minority, so that eliminates a lot of possible scholarships.) | If you're a dependent of your parents and won't qualify for financial aid, then see if you can declare yourself an independent for financial aid purposes. I'm not an expert on this but you can Google it. This would change your status so that your parents' finances aren't factored into the calculation. Besides financial aid, there's merit-based scholarships. Schools differ in how much they give merit-based scholarships; some don't give them at all, while some (often state schools) do give substantial merit aid to attract students who wouldn't otherwise attend that school. The better your GPA and test scores, the more likely you'll get this. Look up colleges you want to go to and their stance on merit scholarships. Barring either financial aid or merit scholarships, you'll be able to pay for college the way most of the country does - through student loans. These usually don't accrue interest until you graduate. You'd make monthly payments on this, usually over 10 to 20 years, to pay off the loans. In general, I also encourage you to be ambitious and apply for the best colleges you can, regardless of ability to pay. Once you get in, then you can use the different offers you get to negotiate a better package from each school. Do NOT psyche yourself out and avoid applying to schools just because you're afraid you can't afford it - this brings your chance of going to the school to 0%. | |
2/7/2021 | Extracurriculars | For athletics, I quit XC this year because of COVID, but I received my black belt in taekwondo during the pandemic. Would colleges see that as me focusing my attention and developing a spike? I also helped found a group that helps my local library run reading competitions, but quit Spanish Club. Is it okay that I'm quitting extracurriculars so that I can focus my energies better? | Every high school student has to deal with covid, so moves that would otherwise look unusual make perfect sense in this environment. In a covid environment, it makes perfect sense to pause group activities thare are on hiatus and increase solo activities. Even without covid, it's OK to change your activities partway through, as long as you're able to justify it to yourself based on a lack of interest or progress. You don't want to just flit from activity to activity without giving it a real try, but once you've given it a try and know it's no longer relevant for you, it's fine to change it to something you enjoy more and will achieve more in. | |
2/7/2021 | Extracurriculars | Second right now I am just a freshman in high school and I am looking for research areas where I can demonstrate my interest in dermatology. I have already found a doctor I can volunteer for . So can u help me find some medical/ dermatological research programs in Arizona I can apply for so I can explore my passion more and build a spike. | Google is your best friend here. Try searching "medical research program high school students arizona" and go through the top 100 links. That will do a better job than any human can. | |
2/7/2021 | GPA | I am a high school freshman in the second quarter of school. Unfortunately, due to COVID-19, my mental health in quarantine caused my grades to drop significantly in my first quarter of this school year. I received three As, two Bs, one C, and one D. This was a major shock to both me and my family, as I have never received lower than A grades as a student. I read your 4.0 GPA guide, and since then I have been working hard this quarter to pull myself back up and regain my usual grades. As a result, I am now confident I will be able to finish off this quarter with all A grades. However, I am concerned about how this will affect my GPA. I was wondering if I need to take all AP classes for the next three years to get my GPA as high as possible and recover it from this first quarter? | As I say often, freshman year is the least important year of your college application. It's a big adjustment period for many, and covid made that worse this year. Colleges care more about your trajectory than where you started out. If you had a rough start but then are a rocketship from here on, you'll be fine. Colleges care most about what you did as a junior. As for AP classes to recover - don't overextend yourself for the sake of making up for a bad quarter. You can be ambitious, but don't take on too much, since it might backfire and your grades will lower again due to being stretched thin. | |
2/7/2021 | Mental Health | One of the things that scare me most about my future is how ADHD will mix with my academics. I was diagnosed with ADHD at a very young age and was recently reminded of it at a particularly turbulent trial session with a therapist. I'm not one to use ADHD as a crutch, but ADHD is one of those things that loves to involve itself in every aspect of my life. When I think about taking the ACTs, I always wonder how having ADHD will come into play with the whole process. I should tell you that I've tried taking medications to cope, but they either haven't seemed to work, or the side effects were too detrimental to continue taking it. I have found ways to slow my brain down during tests by reminding myself to re-read questions, focusing on the test and nothing else, etc. These have had mixed results. I want to know what you think would be a good way to cope with ADHD while preparing for and taking the ACT. | I'm definitely not an ADHD expert nor a doctor, so I don't feel qualified to answer this specifically. I can say three general things: 1) Mental disorders are super personal; the same treatment to two different people can have very different outcomes. It can also take a long time to find the best balanced treatment. Find a psychiatrist you like and trust and work together over the long-term. 2) I'd suggest finding other people who struggle with the same things. I know it's hard to talk about this stuff online but there are more people than you think in your situation, and everyone's hesitant to find each other (I get multiple questions about ADHD). You might post on a place like Reddit, like in the ADHD subreddit (https://www.reddit.com/r/ADHD/) or at the very least read through related posts. 3) Colleges are understanding about mental health, and they know that mental disorders are out of people's control. They care more that you made the most of your situation, whatever that was, and showed the character traits that will let you succeed over time. They won't expect you to have as strong of an application as someone without ADHD. Hopefully this takes some pressure off. | |
2/7/2021 | Tests - SAT/ACT | I'm a freshman in high school. When should I start ACT prep? I know I'm early to the party but believe me when I say, this has been a long time coming. I just want to know when a good time to start preparing is. If you can remember, what was your study schedule like from the time you started preparing? If you cannot recall, what would you recommend? I am talking about how many days a week and how many hours per day. Just want to know if I need to clear my schedule for half a year next year. | I'm a big fan of getting the test out of the way early. Once you get a score you're happy with, you can focus on coursework, APs, and extracurriculars. I personally probably started studying lightly for the PSAT as a freshman and then got my final SAT score in January of my junior year, which made the rest of junior year and senior year stress-free, as far as test prep goes. Each person's study schedule depends on 1) what they're scoring today, 2) what score they want to get, 3) how quickly they improve their score. So I can't prescribe something specific for you (that's what our tutors and program do at PrepScholar). In general, I recommend you start with taking a full-length practice test and seeing what your score is. Then review your mistakes, figure out what skills you need to improve, study for say 50 hours, and take another practice test. This will give you a sense of how quickly you're improving and how much more work you need. | |
2/7/2021 | Tests - SAT/ACT | I was wondering how quickly I would be able to raise up my score. I am a sophomore in high school and my score on the SAT is around 945-950. I want to be able to raise it up to a 1500+ because I want to get into selective schools. I just want to know, do I have enough time to raise it up? How much should I study everyday? I plan on taking the SAT three times. One in October 2021, December 2021, and somewhere in spring and/or summer of 2022. | Since you're a sophomore, you'll still get smarter over the next year and naturally improve your score. But I'll talk as though next year you're scoring say a 1000. A 500 point improvement is not impossible, but it requires sustained effort, strategic studying, and maybe even obsession. You're currently a little below the national average SAT score, and your goal is to leapfrog the majority of students to a 99th percentile score. I say this not to be discouraging but so you realize the magnitude of the task, which is motivating in its own way. If you really want to get to a 1500, I suggest for a period (say a month) you study as much as you possibly can - free up time where you don't need to spend time (on social media, watching Youtube); then spend every spare moment you have on SAT prep. Every typical high school student has at least 10 hours per week they can carve out like this. Do that for a month, take another practice test, and see how high your score improved. Then you can calibrate how much more work you need to do. | |
2/7/2021 | Extracurriculars | I am in a rather unique position in terms of ability to enter an Ivy League school, especially considering the country I'm in (Singapore). Currently 17 years old, I did 2 remote data science internships when I was 16, while attending school. One is with a large govt-funded AI-focused organization, the other with a risk management startup. Right now, I am doing data science research with [technical university] (will be co-authoring journal papers). My "spike" is data science/programming and ability to creatively use them. I chose to pursue my passion further, instead of focusing on exams. This meant my exam results aren't terrible, but aren't impressive either. This is rare, because Singapore is a highly academic (grades) focused society, so the path I chose goes against societal norms. Therefore, it has been hard for me to seek advice on college admissions because they don't have much experience with students of my background. Questions: 1. Are internships with companies valued by admission officers? Should I do another? 2. Given the above information, what do you think are my chances? 3. I'm thinking of starting a blog about data science, finance and economics. Do you think this is worth the time? 4. Since I have roughly 2 years before applying for college, what do you think should be my next steps? (Note: I need to take A-levels next year) | 1. Anything that showcases your personal interests and capacity for achievement is valued by admissions officers. This is true whether you intern in a research lab or a corporation. Similarly, though, the most important thing about an extracurricular is what you actually achieve in it. An internship at Harvard where you're just washing lab dishes is less impressive than an internship at a local college where you drive your original research program and publish papers in reputable journals. Many high school company internships don't give you that much responsibility (because, compared to professionals, high school students typically just have too underdeveloped skills to really be useful to the company). If you are a real hotshot though, and comparable to a college student engineer, then you'll be in a good place. You should always keep pushing for more responsibility and try to carve out projects that you have a major role in. 2. Because I'm a big proponent of building a spike to get into elite colleges, you don't have to be perfect. They do expect that, if you're really as good an engineer as you will portray, math should be fairly trivial and you should get near-perfect scores on math/science without too much effort. It's OK to be weaker in humanities. If you have all-around bad test scores, then it won't invalidate your extracurricular accomplishments, but it will look weird since it doesn't fit the usual archetype of the science/engineering-heavy student. 3. It's worth it if you enjoy writing even if you get 0 visitors. Don't do it if you're just doing it because you think it looks good. I'm a big fan of writing and content creation, but the best content creators love doing it even if no one's watching - this is the determination you need to make it through the troughs when you're not getting the traction you hope for. 4. This is too broad a question to answer without more details, but I think you're on a good track. Keep exploring your interests, use your time judiciously to move you most effectively toward your goals, don't neglect the base responsibilities you have in maintaining grades and test scores. This is a generic answer but there is no real secret to this stuff. | |
1/17/2021 | Coursework - APs | I'm a sophomore in my school's Biotechnology Institute. However, in this program, there are few opportunities to take APs because of the specialized curriculum. Will this lack of APs, hurt me in the long run? I know that you took 12, and I have taken 0 so far. I have plans to take AP Spanish and maybe AP Biology if the program allows it, but I would still have a low number of APs. | Colleges understand that students in different environments will have different access to opportunities. What they care about is whether you maximized the opportunities available to you. Furthermore, college admissions readers who focus on your region will know your program well (if your school regularly admits students to colleges you want to go to, then this is even more true). I'd also expect that your BTI program would have coursework that is as equally rigorous as APs, so it might not be a disadvantage. Another option is to take AP exams even without taking AP courses. If you get a 5 on an AP test, you get 'credit' for that subject even if you didn't take the subject in school. | |
1/17/2021 | Extracurriculars - Research | I'm a sophomore in my school's Biotechnology Institute. Though I am in the BTI program at my school and plan to pursue a career with biotech branches, I have not gone out of my way to focus on science fairs, science-related clubs, or other science things like you did. I fear that I am too late to get involved in this stuff, and I dread doing so because I have no care for science fairs and otherwise. I would love to do an internship with some doctor or researcher who is well-renowned in my area. My biggest struggle now is that I do not know of any good programs in my area that can be my spike. Since we are both from California, would you happen to know of some possible choices? I also have plans to apply for RSI. Would you be able to dig up your application for it or share the main things you mentioned in the research paper, science fairs, and previous scientific work categories? | It's fine not to do science fairs and competitions. Go where your interests drive you. As for choosing research programs, Google is your best friend here. Try searching "research program [your city or state] high school students" and go through the top 100 links. That will do a better job than any human can. You can also do research without a program. Read my previous answers for tips on how to approach researchers for a research position. On RSI applications - read my previous answers for tips on how to apply to RSI. | |
1/3/2021 | Extracurriculars - Spike | Advice on how to score a *1* in the academic rating when applying to Harvard. I know that you gave some examples on PrepScholar, but I was wondering if you could give more, especially for the people who gravitate towards the humanities. More specifically, how could someone be a “ Genuine scholar; ...combined with unusual creativity and possible evidence of original scholarship.”? I am really interested in the humanities, especially human rights, and medicine, especially plastic reconstructive surgery. | One way to answer this is, "what is the best person in the world doing in your field?" You should then try to do a reduced version of this that is still world-class for your age. For instance, students who want to go into science research will look up to Nobel-prize winning research; their version of this is to do meaningful research, possibly author a notable paper in a strong journal, and articulate interesting research directions they want to pursue. Students who want to start a business probably look up to people like Elon Musk; their version of this is to start and grow sizable companies in high school and ravenously learn the skills they need to do so. Apply this approach to your own area of interest. | |
1/3/2021 | Extracurriculars | I am an international skier and have actually won an international race this year, but I am losing interest (after sixteen years)in the sport. I would rather focus on school and setting up a company that I have been working on for a little while. I guess that what I am asking is will the university still recognise these achievements in my application even if I no longer compete in the sport? | Yes, even if you stop doing an activity, they'll still look at your achievements as indicative of your general potential for achievement. Colleges expect you to change your life path during college, so it's less important what exactly you do before high school and more important demonstrating what you're capable of, once you find your true passion. You _should_ prepare for questions on why you lost interest, why you worked so hard for something and then let it go, and why you like your new thing more. You won't necessarily need to address it in your essays, but it might come up in your interviews. | |
1/3/2021 | GPA | I'm in the 10th Grade and I have really bad anxiety when it comes to school. This has caused me to miss a lot of days this year (and my school isn’t even doing in person classes, it’s all online so yeah, it’s pretty bad). Now, since I have missed so many days, I have been really behind on the work and my grades have dropped, and so has my GPA. My GPA last year (9th Grade) was a 4.0 GPA and I never dropped below an A in any of the 4 quarters. This year, the highest grade I have is a C I think. Will this hurt my chances of getting into top schools (like the Ivy League)? | Colleges are certainly sensitive to mental health concerns, so you don't need to feel defeated if your application doesn't measure up to students who didn't have the same setbacks. What they really care about is whether you made the most of your situation and opportunities, whatever those are. There are a few ways to discuss this: 1) applications have a "special notes" section expressly to discuss personal situations like this 2) your recommendation letter writers can speak to your mental health struggles and how you made the most of it 3) consider writing about your lessons and development in your essays. You wouldn't want to use it as an excuse, as you say, but rather focus on what those experiences meant to you and what you learned I am not a mental health expert and I suggest you get professional help on this. A good therapist or psychiatrist can help you far more than your parents or friends or the Internet will. | |
1/3/2021 | Extracurriculars | I want to go to school to study Business/Finance, and I was wondering, what kind of achievements can I get relating to Business/Finance that will look good to get into the top schools (like Harvard and Ivy League)? I know that obviously running a business (and a successful one) can be one good thing, but anything else? | Focus less on what will look good to colleges and focus more on what you're genuinely interested in. The most accomplished people aren't doing what they do merely to look good; they're doing it because they are passionate about their thing to an extreme, and they cannot find any possible better use of their time than to obsess about improving at that thing. They become strong college applicants merely as a _byproduct_ of that passion. In your case, there are lots of avenues to explore, from starting a business of some kind to building web apps to teaching business principles on Youtube/Tiktok to developing your own stock trading software to writing analyses of IPOs. Any of these 'look good' with a significant amount of achievement and passion. But it'll be hard to persist through the obstacles and hard times unless you have a genuine interest in it. And, as a general note to everyone, it's totally OK to not be obsessed about something to the degree that the 12-year-old grandmaster is obsessed with chess. It takes a long time to figure out what you're passionate about, and many adults end their careers never having found it. Still, you should explore broadly and work hard at whatever is your top option at the time. | |
1/3/2021 | Tests - SAT/ACT | I read your method on how to get a perfect score and I think I am motivated enough to commit hard work and effort to it. How many hours a night or weekly should I study consistently? And how many weeks or months might this take considering I put meaningful time to studying? | There is no common answer here; it depends entirely on a few critical personal factors: -your starting level -your rate of improvement In turn, your rate of improvement depends on how much you've prepped before, your learning strategy and general academic ability. For instance, someone who starts at a 28 and has never prepped before will probably find it easier to improve than someone who starts at a 28 and has already prepped for 1000 hours. Likewise, someone who preps strategically, focusing on weaknesses (as I recommend in my perfect score articles) will improve faster per hour than someone who studies unintelligently (eg by doing tons of practice questions without any review of mistakes and correct answer choices). For anyone new to these tests, I'd recommend the following: -take a free full-length practice test to get your baseline score -study for 100 hours, drilling your weaknesses that you identified from your practice test -take another full-length practice test to see how you improved This trajectory will help inform you on how much more work you'll need to hit your target score. There are a lot more nuances (meta test-taking strategy vs specific content skills, diminishing marginal returns as you study more) but this is a good start | |
12/20/2020 | General | I'm a high school junior. I've been reading your articles and I thought that maybe writing you an email would be a good idea, since now I see most of the advice I get is misguided. Just thinking of what I have to do to get into a really good college since sophomore year has been a terrible factor regarding my mental health. I want to pursue a medicine career. I have a competitive attitude, even if I don't quite like it, I really want to get into a top school. Here's the thing: my problem is time. The funny thing is that my problem is all the time that I lost that I'm now missing. What stresses me out is that I didn't lose the time because I spent hours watching netflix or something of the sort. I'm from Dominican Republic, and even if I had always been in the best schools in the DR, they're not compared to the ones in the United States. I moved to the US in my sophomore year. Since middle school, I had always been one of the best students in my math class and in most of my other classes (no, my intention is not bragging, I just try to be honest regarding my situation). However, the education system in DR is different, there are not such things as Honors, Advanced or AP classes. There are just normal classes in each grade and people who are really good students, packed together with the bad students. Given this, when I enrolled into sophomore year in my current high school in the US, the administration couldn't place me into honors classes instead of CP classes. Most of my teachers agreed that I had the potential to be in Honors classes, but that's not how things work. You have to be in CP classes, then level up to Honors classes, then go into AP classes, each of them lasting 1 year. In my sophomore year I felt like I had too much free time; my classes were too easy for me, but I couldn't spend that time either on volunteering work, for example, because I wasn't old enough (once again, DR system is different, and I'm a 15 year-old junior because of that). I did my best and got into all Honors classes this year, except for english, I'm in an advanced english class. However, in my High School, advanced classes are not weighted when counting the average GPA, Honors and AP classes are (10% and 20%, respectively). I have an excellent grade this year in almost every class, I'm hopeful I will get at least a 4.0 GPA this year. I'm also hopeful that most of my teachers will recommend me to AP classes next year, at least 6 or 7. I feel like my grades aren't the problem, and they are certainly not what makes me so worried about my college application. I'm supposed to take the SAT test around April or May of next year, and that obviously worries me since I have to practice a lot, but it's still not the worst thing, because I know if I study hard enough I can get good grades on the test. It's just depressing for me to see how many of my American classmates totally wasted their time in their earlier High School year, not even wanting to pursue a college career. I feel like I'm missing the spike. I'm missing that good thing on my college application, and I wish the only important factor was grades. Since I couldn't do volunteering hours last year, I wanted to do them now, but the pandemic came to be. I feel like I'm just losing time and I won't be able to do stuff because of the new limitations everywhere due to the pandemic, like extracurricular courses. I know myself, and I know how tough the immigration process was for me. I left everyone behind and was forced to improvise a new vision of what I want to do with my life here, in a totally different society. And even knowing that, I still consider myself lucky that at least I could acquire a legal status in the country. I know I have the potential to push myself harder, I know I would do great in a good school, but the colleges don't know that, I have to prove it. They will see me through a paper; an essay, and my academic achievements, and I'm afraid I didn't have the time that everyone else had here to do what I wanted and build my spike. I was never an athletic person, to say something, and even if I was I wouldn't be able to join an advanced sports team because I didn't try out in an earlier grade. I wish I could've had the time to do research programs in local colleges during summer, but I only have one summer left before senior year. It makes me feel so, so "weak", that I could do great things and I can't because I didn't get to this country earlier. I don't have enough money to do things everyone else does, like pay for a super expensive SAT coach or enroll in really good biology research courses that actually cost money. I don't want to regret my entire life that I didn't get into Columbia, or Princeton, or Johns Hopkins, only because I didn't have the time everyone else had, even though I'm putting all the effort to school that my body and mental health allows me. And I'm afraid that, with the pandemic, the time I have left before applying to colleges and my financial possibilities in mind, I might not have the time to better my academic profile. Or even what it takes. | You're clearly facing a lot of anxieties and regret about your situation. I think my best response is to repeat two general principles: 1) Colleges evaluate you based on your personal situation and the opportunities you had available to you. They do NOT compare you on the same playing field as the wealthiest, best-prepared student in the country. Colleges will totally understand that, as an immigrant in your high school years, you'll have faced heavy setbacks compared to people who grew up in this country. They'll adjust your application accordingly. The important thing is that you make the absolute best of what's available to you. Don't sit around pitying your situation; spend that energy thinking of new opportunities and chasing them. As the Serenity Prayer says, have "the serenity to accept the things I cannot change, courage to change the things I can, and wisdom to know the difference." 2) Life is a marathon, not a sprint. Getting into a great college does not set you up for life; I had plenty of bozo classmates at Harvard who are aimless or miserable right now. And people who get rejected from their dream schools can still knock it out of the park and exceed their colleagues who went to better schools. The college you go to matters less than you think. It's more that good colleges concentrate people who have the work ethic and personal traits that lead to success; those people, when put into any other college, would have succeeded all the same. It's hard to have this perspective at your age with your limited set of experiences, but just take away that each day, you just need to try as hard as you can, make the most of your opportunities, deal with your failures, and keep getting better. That's about all that any of us can ask for. | |
12/20/2020 | Graduate School | I loved your guides and was wondering if you would ever make a guide for MD, PHD, or MD/PHD admissions. I find that there is a lack of qaulity information of how to be successful in these endeavors. I have a bunch of questions, but really I would appreciate a general guide to the world of MD/PHD and graduate school admissions. Quite frankly I am a confused high school senior who wants to set himself on the best path. Some of my most pertinent questions are: what do MD or PHD admissions officers look for? Is it the same as undergraduate admissions? How do I do original or novel research (how do you come up with novel research ideas)? | I do intend to write guides to this, but find myself short on time! Compared to college applications, graduate program applications are similar in some ways and very different in others. I'm talking generally about PhD, MD, MBA, JD degrees, as well as becoming a vet, pharmacist, optometrist, dentist and so on. The main difference is that graduate programs want you to have high conviction that you want to practice that profession. PhD programs train scientists; MD programs train doctors. If you're not interested in becoming a doctor, no MD program will admit you. In contrast, colleges don't expect you to know what you want to do with your life—this self-discovery is part of the college and young adult process—so colleges care just about seeing general achievement, not necessarily in the field you'll end up working in as an adult. Materially, this means that your application needs convincingly demonstrate your interest in the profession. This often means real working experience in the field, or at least exposure to it. After all, how do you know you want to be a doctor if you've never observed a patient interaction? So PhD programs more or less require prior research experience, MD programs more or less require exposure to medical practice, and so on. The degrees do place different emphases on GPA and test scores. From my anecdotal observation, -PhD programs don't care much about GPA and GRE test scores (though these need to be high enough to show you're competitive - if you want to be a PhD in math, the GRE math should be trivial for you). PhD programs care MUCH more about your interest and skills in research and prior research experience. In other words, a perfect GRE score gives you very little edge over a good enough GRE score. It's also better to take more advanced, graduate-level classes than it is to earn a perfect GPA on easy classes. -MD programs do care about GPA and MCAT scores, to some extent - there are 'soft' cutoff scores (for Harvard, getting a 95th percentile MCAT score is 'good enough'). They care more about your dedication to medicine, how you proved it to yourself, and whether it's genuine. There are a lot of people who think they want to practice medicine but really are just fooling themselves (myself included). There are lots of routes into medicine, from a typical science background to a nontraditional background like philosophy, the arts, etc. In other words, someone with a perfect GPA and MCAT but little conviction for and exposure to medicine will not get in. -MBA and JD programs use GPA and test scores more strongly than MD and PhD programs do, in my subjective 'feeling'. Empirically this shows in the average test score percentile in top programs - Yale Law School's average LSAT score is a 99th percentile score, while Harvard's PhD programs tend to see average GRE scores around the 90-95th percentiles. I also sense that MBA/JD programs require the least amount of prior experience in the field, though having zero experience does raise question marks about your seriousness. | |
12/20/2020 | Extracurriculars - Spike | I have been following your posts and you speak a lot about a "Spike". I have been a National Public Speaking Champion, National WarCry Champion and 4th in a National STEM Innovators Challenge. I also have always been in the Top 2 for every Model UN I've been a part of. I have also placed in the Top 3 in a pitching session at the Global AgriTech Summit. I am now in the middle of launching [removed]- the [removed] which already has UNICEF and some of the nation's leading youth-led organisations. I am also one of the leading coordinators of a working group under the youth wing of the UN Framework Convention for Climate Change. What scares me is that having too many spikes may as well make me a well-rounded candidate and not anything remarkable. As someone who either wants to study law or environmental science and public policy, I don't know how to position myself in a way that is 'spike'y. All of my achievements have happened even considering the fact that my family is from the bottom 40% of society and my mother is a single parent. Do I position myself with my environmental efforts and public speaking skills? | When I say aiming for too many spikes is bad, that's usually under the assumption that it's very hard to be truly world-class at more than one thing (because the competition is tough and often single-mindedly focusing on doing well in that one thing). When people split their attention among too many areas, it's hard to get the level of practice and development that makes you world class. But having multiple world-class spikes is not a bad thing, in the unlikely case you can achieve it. Let me put it this way - say there were some magical unicorn high school student who was at once an Olympic-level athlete, a chess grandmaster, and started a worldwide nonprofit all at the same time. That person would certainly be stronger than an applicant with just one of these achievements. It's just very hard to do this (again, because the competition is tough). In your case, I sense there are certain things you have a natural interest in (public policy), and other things that simply come easily to you and don't take you much effort to achieve well in (science, hobbies). That's totally fine - talented people often can do well in their side interests without really trying. What I suggest is packaging together your true natural interests into a coherent story. Your pitching/public speaking/UN awards are not individual spikes - they all serve to further your single big spike, your passion for ____. Likewise, my spike was my interest in science research and medicine; doing decently in math competitions, being concertmaster of an orchestra, etc were just side pieces. Finally, this is hard to say without sounding arrogant, but your country is on the smaller side (~30 million in population, compared to 400 million in the US), and is considered a developing economy. Thus, the level of competition is lower than it would be in the US. Your national rankings are still nothing to sneeze at and I'm not at all taking away from your achievements - but be aware that it doesn't equate exactly to public speaking champion in the US. I don't intend to make you feel discouraged by this; if anything, use this to push yourself to achieve even more so that you can become world-class under any ruler. | |
12/20/2020 | SAT/ACT | I am a sophomore and I took the December SAT and I got a 1540(800 in Math and 740 in reading). Since I have about 2 years left, should I take the SAT again? In your opinion, is it worth it? I am aiming to get into top schools. | Congratulations! You're well ahead as a sophomore. A 1540 already is not going to disqualify you from any school, especially if your narrative as an applicant is in science/math. If improving your reading score by 20 or more points doesn't take an inordinate amount of time, I'd suggest lightly trying to improve it. It should never displace time you're spending on academics and extracurriculars, so don't obsess over it. You might promise yourself that you'll get ___ score by ___ date with ___ hours of work, or just move on forever. | |
12/20/2020 | Coursework - APs | Is it better to take regular Physics or AP Physics during HS years ? I have to take it senior year as i thought u have to have a completion of Algebra 2 first, but for my school you can take it concurrently which i didn’t know. And in regards to math, if there’s no way for me to take Calculus senior year but i can take Pre-Calculus can i take Calculus as a math course in college? | As far as applying to colleges go, all else equal, an A and a 5 on an AP course will of course be better than a regular course. The more important question is how well you'll do on AP Physics, and what it'll cost in time and effort. If taking AP Physics would get you a B- and a 3, then you might be better off in regular physics and investing that time elsewhere. Yes, you can take Calc in college. You can take nearly every class past the basic high school level in college, actually. | |
12/20/2020 | Medicine | What’s the best major for students pursuing in the medical field, there’s not a specific major but what STEM related majors is the best? Does the major HAVE to be what your going to pursue in the future or can it be something you're interested in? For example, i like Fashion Apparel Design but i also like being a Medical Doctor too and wanna try both but i’m not sure how to pick. | As I wrote earlier, medical schools don't have a strong opinion of what major you took in college. At Harvard I had classmates who did a more standard bio/science track in college and plenty who studied philosophy, English, and the like. We had a Navy SEAL in our class too. Medical schools mostly care that you are 1) genuinely committed to medicine, 2) can handle the academic requirements well. The minimum you need to demonstrate the latter is a good MCAT score and pre-med curriculum. Beyond that, medical schools seek a wide diversity of students; about the only thing they have in common is their passion for medicine. | |
12/20/2020 | Applications - Transferring | I was wondering if you could shed some light on transfer admissions. Harvard, Stanford, MIT, Yale, etc. accept maybe 30 people per cycle. What does it take to get in, specifically on the extracurricular side? | I don't have a lot of experience in transfer admissions. I do know the application rate is pretty low, possibly lower than the normal application rate, simply because there are so few open slots. My intuition is that you need an application similar in strength as one that would get you in as a freshman. It probably also helps to have a narrative around why you didn't do as well in your first round of applications, but now you're on a much steeper trajectory of improvement after you learned __ or got past ___ obstacle. | |
12/20/2020 | Extracurriculars - Research | I recently read your PrepScholar article on your Harvard application. You said that you got accepted by RSI, so I'm contacting you to see if you have any advice for RSI applications. I want to apply to RSI this year and I haven't done this before so I desperately need help. Could you share your extracurriculars and essays you applied to RSI with, and the RSI interview questions? Thank you so so much! | Applying to RSI is like applying to an early, specialized form of college - the ideas I wrote about in applying to Harvard by being world-class apply to RSI as well. Among thousands of smart, academically capable students, they're looking for people who have a genuine spirit for discovery and research, and often a bit of that rebellious MIT hacker attitude. I don't have my RSI application anymore, and I actually forget whether we even had an interview - sorry, it's been over 15 years! I know that what probably got me in was 1) my prior research experience (in swarm robotics and computer science), 2) being ranked nationally in the US Chemistry Olympiad. My teacher recommendation letters probably helped too. The best advice I have is to explore your interest in research and demonstrate your commitment to it. It's best to have prior research experience, but barring that being a standout in academics can be enough, if you communicate your deeply-considered passion for your subject area. The best mindset to have is less "I want to apply to RSI because it's prestigious" and more "I want to go to RSI because it's the best place to develop as a scientist and researcher, which in turn prepares me to make ____ impact on the world." | |
11/29/2020 | Extracurriculars - Research | I am a Freshman this year and currently have a 3.8 unweighted GPA and 4.11 weighted GPA. I still have 1.5 months so I hope i can fix my GPA. I am taking Alg2, Honors chem, AP CSP, World History, Eng1 and honors Spanish. I participated in science fair all through middle school and won awards each year, but I do not enjoy it anymore as I don't think my time spent was productive. I am interested in neuroscience and cancer research and would like to publish papers and/or do independent projects. I need help to get started - I need advice on where and how to start. Is it possible to get a mentor? | The best way to get research opportunities at nearby colleges is to demonstrate 1) your interest and 2) your capability. For your interest, you should be interested specifically in what the lab is doing and have done your research on what they work on, then articulate why that interests you and what you would like to work on. For your capability, the point here is that most PI's don't want a student who wastes their time and isn't useful in pushing their research forward. You won't be as useful as a grad student, but demonstrate that you know the basics and are willing to learn really quickly, and that you'll stay committed for a long period of time (and not just for 3 months). Basically once you research a lab, you just need to write to the professor, or a grad student you're particularly interested in working with. Customize each reachout; don't just send 200 people the same email. | |
11/29/2020 | Tests - SAT/ACT | How important do you find the act/sat writing portion? Should I take it again even if I have a perfect ACT score? | Some colleges still require the essay section, and some highly recommend it - here's a list we assembled at PrepScholar: https://blog.prepscholar.com/complete-list-which-colleges-require-act-writing-all-schools Check your list of schools to see what they recommend. If none of your schools say they need it, then taking it isn't necessary once you have a perfect ACT score. | |
11/29/2020 | Extracurriculars | Currently I opened an online shop and have made around 2,000 dollars and I am donating all the money to charity. I was wondering, how much money do you think I need to make for it to be for it to be considered a significant part of my application? | I don't have an exact answer, but I think a business with 5 figures in revenue would be pretty impressive for a high school student, and 6 figures would be very impressive and quite rare. I don't have a real basis for saying this, just my own reflection as a business operator of what levels of sales are difficult to achieve. For context, the average Girl Scout apparently sells about 250 boxes of cookies per year, which at $6 per box is in the 4-figure range. The nature of what you're selling is also important—if you're selling novel, interesting items that indicate your passion and creativity, that'd be much more impressive than reselling goods that other people have made. | |
11/29/2020 | Extracurriculars - Research | The application for the Research Science Institute is opening soon. On the application form, you are encouraged to ask a research supervisor (if possible) to write a recommendation for you. How important is it for RSI to have a recommendation letter from a research supervisor? Would it say it's valued more or less than a recommendation letter from a teacher? I've been doing research at a university where my research supervisor is a graduate/PhD student (so he's not currently a Dr., but training to be one). Is it acceptable to ask him for a recommendation letter? Otherwise, I was thinking of asking his PhD supervisor, a professor, but the professor does not know me well at all. Would it be better I ask the professor to write a recommendation using the feedback from the graduate student? | A letter from your research supervisor (either a student or the PI) would definitely strengthen your application. If you've done extensive research but don't provide a letter, this would actually look a little weird. From what I remember, you submit two teacher recs and have space for supplemental recs, so you shouldn't need to trade off a research letter vs a teacher letter. As for who to write it—ideally, the person who writes your letter is someone who knows you the best and can speak to your personal characteristics. I know that you also wish that you could have the professor's name on your letter, and there's a way to get the best of both worlds—ask your PhD student to write the letter, then ask your professor to co-sign it. | |
11/29/2020 | Life Choices | I work with a group of advanced high school students and I'm intrigued by your story, what you've achieved, the research you've done, the companies you've founded. But I have a couple of questions: --Why did you leave the Harvard M.D. program? --Was it a hard choice after everything you invested getting to that point? --What advice would you give high school students who may have to make similarly hard choices about colleges, careers, who they are and/or the direction of their life? | Great question! I get this question often enough that I wrote a detailed overview of how I discovered my personal life priorities, and why that meant leaving medicine and science for business. https://www.allencheng.com/previous-work/#Why_I_Left_Medicine_for_Business It was indeed a really difficult decision, since I'd spent over a decade of my life gearing toward going to an MD-PhD program, but I'm far happier now than if I didn't make that change. And this was a decision that was right for me, given my personal preferences. It's absolutely inappropriate for a lot of other people. | |
11/29/2020 | Applications - Consultants | Recently I seeked counseling from pretty well-known college counsel youtube and it cost around $250. This person who I saw as very trustworthy was saying that my dance was not a spike even though I have international/national awards because the competitions are not as well known and are a different style of dance (more Pop culture, lyrical, musical theatre) rather than classical ballet. Also, the competitions aren't per see "well known" but if you search up competitive dance competitions, some of the competitions that I have attended are on there. I love dance so I am not sure why it can not be a spike even though the competitions to her looked more cheesy and "pageant-like." What do you think? I found this person very trustworthy as she was famous on youtube but she made me feel pretty bad. Another thing she said was that startup companies do not help much. Colleges do not like career oriented things? that seemed a little odd to me. Right now I own a slime business which seems "dumb" as it is a toy but there are people who make slime well and sell 20,000 dollars worth of it. I do not know, do you think a slime business on etsy is "dumb" and not helpful for college at all? (Allen note: in a followup, I learned the Youtuber was this one: https://www.youtube.com/watch?v=d-IvGtFXNNA) | I can understand how crushing her feedback was and sorry you're bummed out. I found this video where she seems to contradict some of the advice she gave you: https://www.youtube.com/watch?v=d-IvGtFXNNA I'm guessing she says one thing in her videos as general advice, but then is not very consistent at applying it and is still rooted to traditional ways of thinking. I would surface this video to her and ask her to reconcile the advice she gave you with the advice she gave in this video. See what she says. You paid her a lot of money so you deserve this kind of response. -------------- Followup: "So I talked to her again and I still could not really grasp her point. For example, for the dance thing, I talked to a few other counselors and they were like I do not think the type of dance matters depends on how you write about it. I originally was writing an essay about in brief- how I did not have the natural talent for dance and was made fun off but persisted until I won an international award. She did not like that story and said that because my dance competitions were like the type "dance moms" go to, I have to write about the crazy world of dance competition? I was confused because the show is fake and the competition world is not crazy at all. I think I am going to go with my gut and stay authentic in my essay. She then said nothing about my slime business when I approached her and asked, she was like oh I guess it can work even though the first time she said no? Like I am not sure. Other counselors I have talked to seemed impressed with my app, not that I think it is good, but her reaction was only inconsistent. She did seem extremely self absorbed and I was genuinely a big fan of her before so ... I definitely think $250 an hour is super expensive so I thought she was going to give me good advice?" My Answer: At this point I would trust your instinct and what you genuinely care about. Colleges want to know who you really are, not who you think they're looking for. If you write in a way that's true to yourself, you will be more enthusiastic, and that carries across. In any case, would you want to attend a college that actually thought your dance competitions were trashy or that your slime business is pointless? Probably not, regardless of its prestige - you want to be at a place that celebrates you for you. I'm sorry you had a poor experience with this person. I'm sure she has a reason to believe what she does, but she also sounds kind of jaded about college admissions in general and that leads her to giving bad, contradictory advice. | |
11/29/2020 | High School Choice | I find your blog very informative as a freshman who is a professional instrumentalist looking to apply to the Harvard/NEC dual degree program. It is so kind of you to share your knowledge with all of us on your PrepScholar blog - thank you! It has helped me so much. I am currently a 9th grader in homeschool (online school) but attended in-person school for all years before 9th grade. When I graduated from middle school, I decided I wanted to homeschool thinking I wanted to attend conservatory in the future, but have now changed my mind and believe that a dual degree program would be a better fit for me. I am currently applying to some of the top private high schools to ready myself for a dual degree program, but understand that open spots in 10th grade tend to be few in number if any. I am wondering if I should apply to repeat a year of 9th grade instead of applying for 10th grade as there will be more spots open? Will repeating a grade negatively affect my Harvard/NEC application or will it actually make my high school career more appealing (because I spent all 4 years in an in-person high school)? | This is a complicated question so let's break it down in a few steps. First, I'd question the assumption that applying as a 9th grader will give you a better shot of getting into those private high schools than applying as a 10th grader. The number of open spots for 10th grade are indeed lower, but the # of applicants is also lower. Also, I'm not sure if they would think your repeating 9th grade is unusual and hold that against you. One option is to ask the schools directly about what they advise you to do. Second, I'd question the idea that you need to attend a top private high school to reach your goal. This goes without saying, but there are plenty of routes to get into programs like Harvard/NEC, including going to public schools. Obviously which high school to go to is a really weighty decision that you and your family have thought about a lot; I'm just giving you some reassurance that it doesn't have to be the perfect plan you've envisioned for yourself. Getting into top schools is much more about what you personally achieve with your time than what school you go to. Finally, I'd wonder about your personal preferences for repeating a grade. Are you neutral about it, in favor of it if it gets you closer ot your college goals, or against it in general? If you're against it, then I would just apply as a 10th grader, since I don't see it affecting your college changes too much. | |
11/15/2020 | Tests - SAT/ACT | I am interested in knowing whether at this point in time there has been a discernible drop in the Colorado state wide testing scores at high school and below. I think we are certainly 4 years into, if not 5 years into the legalization of marijuana in the state. I hypothesize that there is a significant drop in sat and statewide test scores. I have done some research and came up with the following article. https://www.compassprep.com/major-drop-in-psat-scores/ I find very little on this subject. The state department of education buries the results in mountains and reams of data, for which one would have to dedicate serious time to meaningfully decipher. My gut tells me that scores on average, for both state education and the national SAT scores have probably fallen fairly significantly. How about an article exploring the facts if the premise is true and measuring….correlating the drop, if any, to the legalization of marijuana. | That's an interesting question. The baseline data here (http://www.cde.state.co.us/assessment/sat-psat-data) suggests there hasn't been a dramatic change - the state is about the national average, as it has been. You seem to be making two assumptions: 1) legalizing marijuana would increase usage among teens 2) using marijuana lowers academic performance For 1), I haven't seen the data but I'd counter that a lot of teenage behavior has a rebellious flavor, and some things are attractive purely because they can't get them. Once marijuana is legalized, then it becomes much less of a forbidden fruit and thus less cool to use it. I'd expect that effect to compensate for wider access. Some data here: https://www.washington.edu/news/2020/07/20/legal-marijuana-may-be-slowing-reductions-in-teen-marijuana-use-study-says/ For 2), I can see how marijuana would decrease motivation. I haven't seen studies on this either, and I'd guess it depends on dosage and frequency. I can't imagine occasional usage (using it once a week) would cause a perceivable change. In general, I'd guess that unless you see a quarter of the teenage population suddenly smoking round the clock, you won't see this show up in SAT scores. | |
11/15/2020 | Applications - Chances | I had a question about college admissions. Before I go into that, I’d like to thank you about your exceptional blog, PrepScholar, for all the advice about Harvard admissions and SAT. Ive been binge reading these sources since Sophomore year, and I think the absolute world of them. So being a Harvard grad yourself, I was wondering what you would think of my admission chances. I applied to Harvard REA, with a 1520 superscore SAT and a 4.6/4 Weighted GPA, where i took over 20 AP and honors classes. However, Id like to major in math, where I took Calculus BC as a Sophomore, then through dual Credit took Multivariable Calc 3 and Differential Equations, including partials, my junior year. I am currently taking Linear Algebra through a Stanford program as a Senior, and next semester will take Modern Algebra. I received straight As (we dont have +-). Id also like to mention that I am the captain of my high school’s varsity swim team, and I reached out to the coaches at Harvard. Because I made my decision to commit to Harvard later, the coaches already gave out their likely letters, however they told me that they would write a letter of rec to go along with my application. On a side note, I also enjoy languages and should i have enough time, I would like to take Spanish as well. I am fluent in Polish and English, and I took AP Spanish as a junior. Also, I would be a first-gen student because neither of my parents went to college before they immigrated here. Let me know what you think. | For the general reader: as with any of these admissions chances questions, my advice is to be ambitious and apply as though you have all the chances in the world. Whether someone like me tells you have a chance shouldn't really affect your thinking. If someone told you you have good chances, you need to avoid sitting on your laurels - you should keep pushing as hard as you can to exceed your goals. And if someone told you you're a reach, you shouldn't be demoralized - you should brush it off and keep pushing. So my opinion is not all that important - it's what you do internally that really matters at the end of the day. For your application - you'll get your results soon so don't worry too much about what I say - I have so little information about the rest of your application that I can't easily predict your chances. This is more for the benefit of other readers. To start, you are academically strong with a special focus on math, and you're now taking college-level math, which is advanced among most students. The first-generation college student story and being competent in athletics is also good. You're in a solid, well-rounded position. Beyond coursework, I would have liked to see more of an extracurricular application of your math talent, whether that's in math research or math competitions (AMC/AIME/USAMO and the like). This would help calibrate you to the top math students in the country. Advanced coursework is good, but not quite as standout as being one of the top in the country, as validated through a competition or research. | |
11/15/2020 | Extracurriculars | I'm a ninth grader at a specialized high school & I've been trying to build my "spike." I've published my poems in several literary magazines & I've published articles regarding serious topics (my most recent one about SARS). When I'm not writing, I am a tribune on the student congress & a member of National High School Model United Nations (NHSMUN). Despite participating in these extracurricular activities, I still feel very perplexed. Therefore I have two questions to ask you: 1. Does one activity hold higher "credentials" than the other? If so, what is your guess? 2. I am a ninth grader, but I don't want my efforts to go to waste. I'll continue to aim higher as I enter 10th, 11th, and 12th grade BUT will colleges truly look at my ninth grade efforts & take them into consideration? | No activity in itself gets more credit than any other - it all depends on your level of achievement. It's more impressive to have a Youtube channel with a million followers in some obscure hobby like lockpicking, than it is to be a simple member of a well-known club like Key Club. It's more impressive to be the nation's top Rubik's cube solver than it is to participate in Model UN as a mere member. Get the picture? You have a lot of interests, and I would try to probe 1) where you have the most intrinsic motivation, 2) where you have the most potential for achievement. The two go hand in hand, since you'll work harder for things you care more about. As far as 9th grade efforts - it's the least important of all your grades, but don't see it as wasted. The work itself isn't as important as the skills you learn and the preparation for bigger things next year. If you work hard now, you'll find better opportunities in 10th grade, and likewise from 10th to 11th, etc. For instance, say you get deep into Model UN this year - you outshine your classmates, and you get a leadership position in 10th grade, then president 11th grade and guide your team to win the local championship. In contrast, if you slack off 9th grade because you don't think it matters, then you'll be in a bad position in 10th grade. | |
11/15/2020 | GPA | I have a very bad GPA . Do i have to explain in my LOR and SOP? | If you have a good justification for why you got bad grades, and why that doesn't represent your potential, then yes you should address this. They should be plausible though and not sound like excuses - a heuristic is that even a very talented, smart person put in your situation couldn't possibly have done well. Common situations include having to support the family, a rough home environment, mental health issues, and so on. You should also try to get your teachers to vouch for you in their letters of recommendation, since their word carries more weight than yours. | |
11/15/2020 | High School Choice | I’m an active duty military Dad of a high school freshman who is an overachiever with a dream to go to Harvard. She will probably have the grades, test scores, and extracurricular activities (music, varsity sports, student government) to be competitive for admission to a top tier school. We are moving next summer after spending 3 years in Asia where my daughter attends the local Department of Defense Educational Activity (DoDEA) school. We have an opportunity to move to Europe for three years where she would continue to attend DoDEA schools or we could move to Washington DC and would send her to well regarded public school. If we had moved to DC last year she probably would have been very competitive to attend Thomas Jefferson Magnet HS but openings for Sophomores are extremely limited. She is concerned that going to the DoDEA school in Europe will hurt her chances for a top tier college due to the lack of an academic challenge and limited extracurricular activities opportunities for her in Europe compared to DC. I believe that her diverse background as a military child, who has changed school systems multiple times due to seven moves, will demonstrate resiliency and applying from a DoDEA school while living in Europe would help her standout from the pack of overachievers who apply. What are your thoughts/advice? | In general, colleges care about whether the student maximizes the opportunities that they had access to. A lot of a child's life is out of their control, including where they live and what school they attend, and colleges understand that. Students are compared more to other students with similar circumstances, rather than to the absolute standard of the best prepared, wealthiest family in the best school district. This is obviously a very important and personal decision for your family, so I can't advise you on whether the opportunity in Europe or the one in DC is better with all thigns considered. But generally, I wouldn't try to strategize around what would make a student standout, such as the general strategy of becoming a big fish in a small pond - it's just as likely that the better opportunities in a more competitive environment might lead to greater overall development, and that your child would shine even brighter in a DC public school. Even if she goes to a DoDEA school with less challenging coursework, nowadays with the Internet, there are more ways than ever to make your own opportunities, regardless of which school you go to. You can take AP courses online, explore personal interests deeply, and collaborate with/reach other people. Ambitious, driven students can do more, from anywhere, than ever before. As part of a military family, she might also have access to programs that civilian families don't. I don't know this area well so I can't recommend anything specific, but I imagine they exist. I don't have a clear conclusion here, but I'd say that if she can develop a world-class application, while being in a DoDEA school with less opportunity, she would outshine a world-class application from a plain family background. But whether a merely strong application in a DoDEA school would outshine a world-class application from a plain background is harder to tell. | |
11/15/2020 | Choosing a College | College is expensive and it seems crazy to pay such high amounts for college tuition if it will continue to be all or partially online. I know that Harvard is close to your heart (as I believe that you are an alumna). If one could even get in, regular Harvard is super expensive where Harvard University (through the Extension Schoo) is a fraction of the cost and much, much easier to get accepted. I do not know if regular Harvard has continued to meet in-person, but at some point, what would be the difference between regular Harvard and extension Harvard (of both are totally online and taught by the same or similar faculty --- with the exception of the tuition difference)? I only use that example as it is probably closer to your heart --- and everyone faces the same feeling. In my case, I went to the University of Maryland College Park and I do feel that there is a difference between the University of Maryland College Park and the University of Maryland Global Campus. But if both are online (and taught by the same or similar faculty)- what is the difference other than your tuition bill? | Caveat: as a graduate of Harvard College I'm biased and I'm going to sound like a total snob. If you see attending college as simply the literal lectures, then of course you wouldn't need to attend college. You could get a Harvard-equivalent education online (through edX or Youtube). Why people still pay $50k per year to go to schools like Harvard is a combination of other benefits that are hard to get elsewhere: -credentialing and selectivity: if the world's top high school students apply to a college and only 5% are chosen, then simply by being admitted to Harvard, you already show that you've passed a very selective filter and thus are likely to be qualified. Similarly, if you hear of someone being an Olympic runner, without knowing anything else about the person, you already know that she's likely a world-class runner. Employers and graduate schools use college credentialing as a trustworthy signal of general competence (though it's not a guarantee - there are lots of bozo Harvard grads - so they use other signals, and the older you get the less your alma mater matters). -peer community: coming from a public high school, I found the Harvard student community to be on a totally different level. Everyone was highly competent in their own ways; some people were literally the best in the world at what they did. Being around these people, learning from them, working with them, and befriending them was highly enriching. -extracurricular opportunities: at places like Harvard you get the sense that you could work on anything you wanted with the world's best people. For example, in science you could find professors doing the cutting-edge, high-impact research and work in their labs. It's hard to do this virtually, or at a lower-ranked school where the faculty teach well but aren't doing the meaningful work in the field. All this applies generally to any top 20 school, and to a lesser degree in top 50/100 schools. Compared to Harvard College, Harvard Extension School is much weaker in all these attributes. It's nowhere near as selective and so it carries weaker credentialing, a less qualified peer community, and less rich extracurricular opportunities. I do expect that over time colleges will have less of a grip on student life outcomes, since there are other ways to educate yourself and get credentialed. There's already been a shift toward de-emphasizing credentialing and emphasizing real-world results. If you learn to code and start companies in high school, there really might not be a need to go to college, depending on your personal goals. These societal trends are slow to change, though, and for the next few decades, colleges should continue to offer the value. | |
11/15/2020 | Extracurriculars - Spike | what should I do to develop a spike in entrepreneurship ? I love entrepreneurship and thats my strong point. | To develop a spike in any field, think about what the world's best people in that field are doing. In entrepreneurship, it's straightforward - start successful companies. That's the best way to show your passion for entrepreneurship. Of course, as a teenager you only have so many skills and resources to start a business, so you can be creative with this. Entrepreneurship takes many forms, including being a content creator, running your own Etsy shop, developing apps, creating new organizations, and so on. | |
10/25/2020 | Middle School | I am a mum of a 10 year old boy (based in UK). He will be applying to secondary schools next year. Maybe its sounds insane I am asking question in a matter of such a young boy but your article created a big impact on my thoughts. Subject in a blog concerned Ivy Leage Universities and creating a big spike, which gives you more chances of becoming a shining star. I was wondering if you would apply it also to the early stage of your academic life (applying to secondary/middle school US). Most secondary schools In UK want your child to be a super-hero in all subjects (maths, english, sience, do well in sports, have lidership skills, playing on instruments etc). What would you suggest me as a mum? How to guide my young boy and what to say at his interview or write on his applications? | Personally I think middle school is a bit early for a student to know what they're really passionate about. The teenager years and high school really change that a lot. If the interests are more academic (as opposed to say sports), then a solid academic foundation prepares the child to pursue whatever interests them as they age. I think this is why your middle schools are looking more for well-roundedness, so they can be shaped throughout school. As for applications for secondary school, I'd first try to be authentic - after all, you ideally want a school who accepts your child for who he is rather than who he's pretending to be. Beyond that, I'd appeal to what each type of school is looking for in an applicant, if they tend to differ in this. | |
10/25/2020 | Applications - Interviews | my daughter is applying to Harvard this year under early action plan. Just would like to have one clarification from you - how the interview for harvard is being arranged, as they asked "Have you arranged for an interview" in thier application form. Whom should we arrange the interview - with professors at harvard or aluminus? Kindly help. | I think this is just asking whether you've for some reason already arranged for an interview. Once you submit an application, they should reach out to you automatically for an interview; you're not expected to handle this yourself. From their web site: "There is no need for you to arrange this conversation. If it is possible to arrange an interview, you will be contacted by one of our alumni interviewers": https://college.harvard.edu/guides/what-expect-after-you-apply | |
10/25/2020 | Coursework | I’m a sophomore who plans to go get into Boston University- but i’m lost. My school doesn’t offer AP classes or Honor classes. In 9th grade, I got all As and Few Bs (regular classes) My only B’s we’re in Earth Science and Geography (which i very much regret) I’m in n 10th grade and these are what my grade look like so far : (i’m taking 2 english courses and 3 science courses) Spanish 2 : 91 English 9/10 : 97 English 11 : 96 Biology : 100 Chemistry : 96 Algebra 2 : 100 Bible : 93 World History : 97 Health : 100 Art : 100 by the way, I plan to start shadowing this summer at my local hospital. I’m worried my grades are not enough to “Wow” then so I have to rely on my SAT score to give me a good application booster. Any suggestions ? | If your school doesn't offer AP classes, that's something that colleges should know when you apply. They want to see that you made the most of your situation, whatever that is, rather than comparing you to the ideal student in the ideal learning environment. Keep trying to push your learning and growth beyond what your school offers - can you take AP classes online? Can you take summer courses at neighboring high schools? Can you study using the AP curriculum and take the AP exam yourself? There are options even if your school doesn't have AP courses. Having a high SAT score will help compare you to other students on an equal playing field if you're missing other signals like AP courses. | |
10/25/2020 | Extracurriculars - Spike | I am currently a senior in high school but I have been reading and following the guides on your website since I started highschool. I am looking to study Neuroscience at Harvard or MIT and although I am in top 10% in a class of 700+ students, I don't have good SAT or ACT scores to send, only AP scores of 4's in English and Biology and a 3 and 5 for Seminar and Research respectively. Since I don't have much time to change anything as a senior, I wanted to know if my "spike" in my school's AP research program along with multiple awards in local Science Olympiad competitions would make me more competitive or less competitive since this is not at a national or international level. I am working to publish my research paper from last year in a high school journal, entering it in a couple competitions and I am currently working on another research paper..all my research has been in Neuroscience so far.. Thank you | As I defined it, a spike is an area of concentrated effort and achievement that distinguishes you from most other high school students. Your AP coursework and local Science Olympiad unfortunately don't set you apart since tens of thousands of students probably have similar coursework and accomplishments. Your neuroscience research, however, sounds promising. If you can take your research into a high level - placing in competitions or getting papers published - that would be a much stronger spike than your coursework. | |
10/25/2020 | Applications - Early Action/Decision | I had a query that in restrictive early decision. I can apply to only one college or more than one college which offer restrictive early decision? I am little confused about it. | There are a few different kinds of early admissions: -Early Decision: If you get in, you have to attend the school. You can only apply to one school this way. -Early Action: If you get in, you don't have to attend the school. You can apply to multiple schools this way. -Restrictive Early Action: If you get in, you don't have to attend the school. You can apply to only one school this way. So REA is like a blend of ED and EA. | |
10/25/2020 | Extracurriculars - Spike | I have a few questions regarding my application and I kind of just want a professional opinion - someone who has dealt with the agonizing process of coming up with a cohesive application for colleges. I started college applications over the summer without much research and after reading your blog posts over your Harvard application and how to get into the top competitive schools, I realized that I will have to restart parts of my application. More specifically, the awards and honors section. For a bit of context, I really enjoy designing and being an engineer and hope to be an aerospace engineer in the future. I really want to attend MIT due to the ample amount of opportunities the school offers, its prestigious status, and the student-centered and initiative/risk-taking culture there. However, I don't quite have any honors and awards. I don't have any national-ranking achievements and don't have any design or computer-aided-design (CAD) related awards (since high-school competitions for designing are very limited). My spike would obviously be the amount of time I've put into robotics - it wouldn't be a stretch to say I've put in 59 hours or so per week - and my determination to learn. I've participated in an internship by NASA (acceptance rate of 13% and receives applications from across the US), but only have certificates of achievement (for 150 hours put into the internship) and other certificates of accomplishment - basically, nothing that is on the level of 6th place in a national science competition. I was thinking, since my spike is centered on CAD, perhaps I could try to receive certificates for CAD (the program I use offers certificates after passing an exam) to show to colleges, but where does that rank in terms of achievements? These certificates are handed out to anybody, including industry-level designers, and have varying degrees of difficulty, from associate, to professional, to professional advanced, and expert. I would really appreciate it if you could give me a bit of your take on this issue, since this is really stressing me out - MIT means a lot to me and I really don't want to lose this chance. If not, that's understandable - you probably receive hundreds of these kinds of emails per day. | First, it's great that you've identified an area of your interest. This is a bigger step than you realize. Schools like MIT have an engineering-minded, "hacker" culture that cares more about what you DO with your interests than about classical credentialing. So you have an interest in CAD - what do you DO with it? What can you create? How can you make an impact with your passion? What quirky things are you so interested in doing that you don't care whether it looks good on your application or not? The internship with NASA is a good start. What can you do beyond this? Can you participate in a real-life project to apply your skills? Can you educate others with your knowledge? Can you build your own cool toys? Let your creativity run wild. | |
10/25/2020 | Applications | this is a rather weird question that i am going to ask you. please if its possible could you tell me your honest opinion. i recently applied to college and i've lied about my extracurriculars (which i shouldn't have done) i've listed two of your harvard extracurriculars as my own .... will i get caught? should I withdraw my application? | Well I can honestly say I've never gotten this question before! I should first state the obvious - that this is unethical, cheats other people out of their real hard-earned accomplishments, and no one should do this. But let's talk about the practicalities, since cheating does happen and we should think about how well it works. In general, I see lying on applications as high risk low reward - it's relatively easy for admissions officers to verify your most important accomplishments, and if you're caught lying it's an instant rejection. For instance, if you say you were an Olympic athlete, it's easy to find out you weren't. They also expect to see impressive accomplishments mentioned in your recommendation letters by your teachers (such as being part of an academic team, winning awards, or being a student leader). The less impressive the accomplishment, the less easy it is to verify, but the less it also adds to your application - for instance, being a member of a club. In my case, if you copied two of my extracurriculars, the best accomplishments are easy to verify (like the chemistry olympiad, going to the RSI research program, or being concertmaster of an orchestra), and the less impressive accomplishments won't benefit your application much. If you did the latter, then you probably won't get caught, but don't count on it making much of a difference in your chances of getting in. I can't advise on whether you should withdraw - that's up to you and your personal standards of ethics. I am pretty curious though about what the result would be if they read your application. | |
10/18/2020 | Coursework - IB | I'm an incoming senior and I've been doing the IB Diploma program for 3 years. However, because of my dad's jobs, we constantly moved, I've attended 3 schools for my high school and this last school just said that I can't be in the diploma program anymore as they have different requirements and classes availability compare to my other school. IB has always been a part of my high school identity and now having to drop it, I've been so sad for this whole week, I feel like my college path just got shut down. How important do you think IB Diploma is to college consider not having it. Also, can you give some brief information about college advices, I'm doing my college essays and SAT now but I'm just clueless, I'm practicing and studying but I don't know where I'm heading. Thank you. | The good thing about college applications is that they take into account your personal situation, rather than just looking at your coursework and credentials in a vacuum. Moving high schools is a jarring transition; doing it twice can disturb any high school student's momentum. Certainly you get leeway for not completing the IB program if a school doesn't offer it. What colleges care about is taht you make the most of your situation, whatever that is. Keep trying to push yourself based on what's avaialble to you. You might consider making your own opportunities (for instance, if your school doesn't offer a particular course, try to take it online.) In terms of your other questions, Google is your best friend. Think of what questions you have, Google it, read the top 5 links, then search for more questions. This will get you very far if you put in the work; many students don't. | |
10/18/2020 | Tests - SAT/ACT | I'm an international student. I'm taking the ACT test in December. If I decide to retake it in March, will I have to send both results to the school that I'm applying to or can I choose which one to send? | Different schools have different policies for test scores. Most participate in a version of Score Choice, which gives you the option of sending only some of the scores you've received. Some require all scores to be sent. I'd make a list of schools you're interested in, research their score choice policies, and then see if that influences your decision. In general, I'd encourage you to take more tests if you're confident you can get a higher score - schools don't really mind your retaking a test if you're showing good improvement | |
10/18/2020 | Tests - SAT/ACT | Are three months enough to improve my ACT score by 5 points? (specifically English and reading) | An improvement of 5 points isn't unreasonable. Whether you can achieve it in 3 months depends on how much you've studied before (the less you have, the more you'll improve), how hard you study, and how intelligently you study. Here's a guide I wrote on how I recommend everyone study: https://blog.prepscholar.com/how-to-get-a-perfect-act-score-by-a-36-full-scorer | |
10/18/2020 | Applications - Special Situations | I struggle with severe depression. I'm currently a junior and know I have the potential to thrive in more prestigious colleges but my current application won't reflect that since I spent the majority of my freshman and sophomore year either in therapy or in my room sick from medications. How would I be able to demonstrate my academic ability and passion for law, while also explaining my lack of accomplishments due to my health history without sounding like I'm using it as an excuse? | Colleges are certainly sensitive to mental health concerns, so you don't need to feel defeated if your application doesn't measure up to students who didn't have the same setbacks. What they really care about is whether you made the most of your situation and opportunities, whatever those are. There are a few ways to discuss this: 1) applications have a "special notes" section expressly to discuss personal situations like this 2) your recommendation letter writers can speak to your mental health struggles and how you made the most of it 3) consider writing about your lessons and development in your essays. You wouldn't want to use it as an excuse, as you say, but rather focus on what those experiences meant to you and what you learned | |
10/18/2020 | Applications - Essays | First off, I would like to thank you for everything that you and your company have done in terms of the college application process. I cannot count how many times I have found answers on your page. I apologize in advance for how long this email is. As a fellow Asian-American, I was hoping you would be able to answer a few of my questions. So, I am a big-time alpine ski racer. I have been ranked top 15 in the country, and placed 15th in the nation (for my age group) at US Nationals last season. I go to a public school in NJ, which I leave from in November to attend a ski academy in Vermont (Killington Mountain School) for five months. I then return to my home school in NJ in April to finish up the school year. I have done this since freshman year and will do so again for my senior year. I believe this is my so-called “spike”. I am also a dedicated scholar, and have maintained a 4.4GPA (w/ a 4.6 Junior year) and straight As. I will graduate with 60 AP credits and I look forward to being able to study biomedical engineering in college. I took the ACT last year with no practice and got a 32, which I was not happy with and I have practiced since and I am confident that I will get a 34-36 when I take it later in October. I have taken a look at your college application to Harvard, which is extraordinary. The issue that I face is that due to my commitment to ski racing, which is year-round with extra training and summer camps to Europe, I have not been able to find the time that I would have liked to participate in extracurriculars. Leaving my school for half of the year presented logistical issues when it came to clubs, etc. as well. I have a strong desire to learn and I have been eyeing top colleges to apply to. Going through my college essays so far (MIT, Stanford, UCLA), I felt confident that my essays would make me competitive in the admissions process. However, when I looked at the Harvard essays, I found myself stuck in a rabbit hole. I feel as though especially after reading your essays, that I simply am lacking the extracurriculars to write about. For my CommonApp essay, I have written about becoming a pioneer. I first start off with struggles during my childhood of being afraid to be different, then talking about an experience that opened me up to the concept of being different and using that to my advantage. I then go on to talk about committing to go to the ski academy for only the winter when most athletes go full time just so I can maintain my academics at a high level (ski academy academics are terrible). This is a difference that I chose, not one that was unavoidable. I then finish off with an initiative I started to bring my school to volunteer at an organization for disabled people (teaching them how to ski) even after facing opposition. This is where I get comfortable being different and do something completely on my own. For Harvard, which is my dream school, I have wrote an essay about creating an electric skateboard for the intellectual activities essay. I am looking to write the extracurricular essay on volunteering at an organization for disabled people (helping them ski), which is where my interest in biomedical engineering stemmed from. For the miscellaneous essay, I was going to write about being an adaptive thinker, whether it be creating games to play with other people, solving problems, etc. I looked back at these three essays and I recognized that these would not help me stand out. Thinking back to your concept of the “spike” I started wondering if I should write about my commitment and deep passion for skiing. I was wondering if there is a chance at me being accepted, and if there is, what I should focus on in terms of the essays. I am very unsure right now and questioning if I should even apply anymore. I am so so so sorry for the long email and I will be extremely grateful if you could provide your insight. I also do not mind if you publish this on your website. Thank you so so much again for everything you do! | It sounds like you have a strong spike in your athletics. As with many student athletes, it's natural that you haven't had time for much other than school and athletics. That's OK - colleges understand the sacrifice, and they don't expect you to be well-rounded in every dimension. That's the essence of a spike - to put in full-throttle effort, you HAVE to sacrifice other things, which in your case means the typical extracurriculars other students have. Don't worry about it. For essay topics - you shouldn't be reading my essays (or anyone else's) and comparing your essays to theirs. You are a unique person and you should write something that is true to you and what you care about. There is no "model" essay to emulate; doing so would make you sound inauthentic. So don't worry that you can't talk about extracurriculars you don't have - write about what you know. As for chances of getting in, whether someone like me tells you have a chance shouldn't really affect your thinking. If someone told you you have good chances, you should avoid sitting on your laurels - you should keep pushing as hard as you can to exceed your goals. And if someone told you you're a reach, you shouldn't be demoralized - you should brush it off and keep pushing. So my opinion is not all that important - it's what you do internally that really matters at the end of the day. I do generally advise that students apply to every school they're interested in; the cost (in time and money) to do so is usually small, but the gain if you get in is huge. | |
10/18/2020 | Extracurriculars - Spike | So I have read your article about to get into Harvard. I really loved that article and believe it is a must-read for any high-schooler. But anyway, my question is that for your idea of your idea of a big spike. Is it a good idea to have 2 big spikes? I am saying this because I plan to try to go to Harvard for college and right now I am only a high school freshman. The 2 big spikes I want to focus on are Biology and Latin. So would this be a good idea? Also how do I compete for getting some good competitions in biology. I want to do this because past achievments can stand out in college and it can help develop a good spike. | Generally, the nature of a spike is usually that you pursue something with such focus that you only have ONE major spike. Consider that if someone had 20 spikes, they wouldn't be very "spikey" and instead would look well-rounded. As a freshman, though, you have time to figure out what your interests are, so it's ok to hold onto multiple options until you really figure out what you want to do. So it sounds OK for now, though do think about what you're passionate about where you can demonstrate that deep achievement and focus. The most prestigious competition in biology is the Biology Olympiad (going all the way up to the International Biology Olympiad). There are also research competitions like Regeneron; these require more than just studying and need you to work on research. I'm not as familiar with Latin, but there are probably analogues of spikes there (think - who's someone you admire in Latin, and what have they achieved?) | |
10/18/2020 | Tests - SAT/ACT | I’m a HS Sophomore in the US. I tend to go through phases of freaking out about colleges, my futures, and grades. I decided to stop being scared and do something about it. I made a plan for my HS years and my starting college years. Right now in my plan, I’m concerned about the SAT. I want to be able to get 1550 or better to give me a fighting chance at MIT. I’ve read your article about how to get a perfect score, and it did help a lot, but I’ve still got some questions. I haven’t done the PSAT yet but I’m going to soon. If I study, I believe the English portion of the test should be easier for me. But Math is what I worry most about. In 7th grade, I made the mistake of losing interest in Math (not knowing it would be important to my career path), and I got set back to Alg 1 instead of Alg 2. I’m still highly capable in mathematics, but I’m missing certain classes I from this Sophomore year that I think would benefit me on the SAT (EX: AP Calc). I just need some advice on SAT Math from someone trustworthy. It’s very important for my career path. I’m afraid this could mess up my whole college plan. Anything would be helpful. | The good news is that the SAT doesn't need math anywhere near as advanced as calculus. It's designed to be a nationally accessible test, which means it doesn't test require math more advanced than standard high school math. So you're not at a disadvantage. It sounds like you're pretty scared of the SAT right now, and I think that's because of lack of exposure. I recommend that you take an SAT practice test as soon as you can. Don't worry about how you do - just take it and get a score. You might do better than you expected. Even if you don't, now you know what the nature of the beast is, and you can start tackling your weaknesses to get a good score. | |
10/18/2020 | Extracurriculars - Spike | I read your article about how to get into Harvard and the Ivy League, I found it really helpful and I really appreciate it. I started thinking about making some achievements as a high school student and I will try my best to develop a huge spike. But I have one question, in the ninth grade I got into the number 1 ranked high school in my country with full scholarship. so, do you think this is an achievement that I can mention in my application? Is this something that universities like Harvard would pay attention to? | Good job getting in! It's a good start, but getting into a high school isn't really an achievement to be listed on your application. If it's a really competitive high school, then colleges already know that; when they see that your application lists that school, you've already "earned" that credit. What's important now is what you do AFTER getting in - you have three and a half years to prove this out. It's the same with college - if you meet someone who says he went to Harvard, that's kind of cool, but you really care what the person has done with his life afterward. There are plenty of bozos who get into schools like Harvard but don't make much of their lives, so the fact that they went to Harvard shouldn't impress you much. Same goes with getting in your high school. | |
10/18/2020 | Extracurriculars | I live in Addis Ababa, Ethiopia. I learn in ---- school and I am in 12 grade. I was planning to apply to Ivy league colleges. Unfortunately, I couldn’t finish some requirements; (i.e, national and regional science fair), due to the pandemic. And my country doesn’t provide the platform for students to do researches with professors and other researchers. So if you could give me some advice, it will help a lot. Thanks in advance. | Lots of students are in your situation, where their lives were upended by covid. Colleges don't expect you to have as strong of an application as if covid didn't happen. They do however care about how you make the most of the situation and the opportunities available to you. Can you demonstrate your interests in ways other than what you had planned? For instance, is there any way to take your interests online, or to complete the projects you would have done for the science fair even if it isn't happening? As you're a senior there might not be a lot of time left, so you might also have to lean on all the work you did pre-Covid to boost the rest of your application, and talk about how you made the most of Covid elsewhere in your application. | |
10/11/2020 | Extracurriculars | I am looking for advice on my big spike, as my spike is not super traditional. My spike is locksport (the sport of locksmithing). As per achievements, I started the locksport club in my school, and helped start not one, but two chapters of international locksport organizations in my hometown. (TOOOL and Locksport International). I have been the president for one of them for two years, and I also offer classes in my areas on locksport, and how to pick locks. In my opinion, this is a good spike. I spend many hours lockpicking every week, and try to bring it to the next level by offering classes, and trying to be a leader in the group. Otherwise, I do have a small spike that is more traditional. I also do Science Olympiad. I have won first in 3 events in local and state competitions in biology-related fields (disease detectives, microbe missions, and anatomy and physiology). | You definitely have an interesting hobby that can stand out. There certainly are not that many locksport practitioners, but keep in mind that uniqueness itself is not enough to be a formidable spike. There are many people with unique hobbies out there - let's say archery, whitehat computer hacking, basket weaving, Irish folk dance, stone skipping, Tai Chi, and so on. If you put all of them in a bucket, how would you choose the ones that are most impressive? A spike is not just about uniqueness - it's a demonstration of considerable effort, talent, and achievement. A good way to show this in a more quantifiable way is either 1) doing well in a notable, competitive competition, 2) achieving considerable impact. You have a good start, but keep thinking about how you can push your passion further, either by honing your craft or by deepening your impact. Could you set up larger conferences for your sport? Could you make cool videos (like Lockpicking Laywer) to attract many more people to your sport? | |
10/11/2020 | Extracurriculars | I have a question. I genuinely have 40 hours of extracurriculars (focusing on two spikes) on top of 6 AP classes. Does that sound unbelievable or like a lie if a college admissions officer were to read that? | Keep in mind that at 16 hours per day and 7 days a week, you only have 112 hours of waking time per week. 40 of these are used in school, probably 10-30 need to be spent on homework (depending on your classes and speed), and some amount should be spent on basic recovery and free time. So 40 hours of extracurriculars is reasonable and not outlandish. It would suggest that you have a very packed schedule, but this isn't too surprising (and I was fairly similar). However, anything about 80 hours would be unrealistic and viewed with heavy skepticism. | |
10/11/2020 | Applications - Chances | Quick question, does creative writing count as an “academic” category for the Harvard rankings? Or extracurricular? | (For background, check out my explanation of Harvard's rating system: https://blog.prepscholar.com/harvard-asian-admissions-lawsuit-application-strategy). I'm not totally sure, but I'd guess that it's extracurricular. Academic has more to do with mastery of academic subjects (physics, economics, etc) and indicates future scholarship in the field. | |
10/11/2020 | Motivation | i've been thinking about the ACT a lot, even though i have 3 years until i actually have to take it i want to be prepared. Growing up nor my mom or dad finished College, and my mother works hard everyday and I see how much physical work she puts in just to provide food for us and I can't help but feel sad because I want to be able to help her out once I graduate College but of course, you need to have good grades to get accepted. I've been seeing a lot of things on the news about people not having the same chances but I have to disagree. There are people who come from India, Asia, the Middle East with nothing but the shirt on their backs and they've become successful. So everyone has a chance, but anyways I'm getting off topic. The reason I emailed you is because I want to know what motivated you and how you felt getting a perfect score, and how you became successful. | You're asking a question about motivation, which is a great thing to ponder. The way I see it, the foundation of motivation comes from answering life's greatest questions—what do I want to accomplish in this world before I die? Why is that? Why do I even need to exist? Once you have this answer, then trace your final goal back to where you are today. What are the steps you need to take to meet your final goal? Once you have this mapped out, you will find the energy and motivation to tackle all the steps in the path to achieving your goals. On the other hand, if you lack answers to these questions, you'll find it hard to summon the motivation to put in the work (since you don't really care about the work in the first place). Of course it's too much to ask to have a really solid answer when you're a teenager (I spent much of my 20's figuring this out for myself), but you can at least have certain things you care about that drive you. It sounds like you care a lot about caring for your parents in return for what they did for you. That's a fantastic motivation. You probably want to end up with a great career in some type of field; to get there, you'll need to go to a good college; to go to a good college, you'll need to do well on the ACT, coursework, and extracurriculars. Personally for me I really wanted to be a doctor/scientist, and that mapped all the way back to excelling in high school. There were other motivations (I loved competing and being the best academically) but the most important was having some direction that I was aiming toward. | |
10/11/2020 | Applications - International | 1) Amidst covid there is rumor going in my country that the acceptance for international students will be extremely low, is it true , what is your take on it? 2) I read your article about spikes, what is the best way to show your spikes considering the covid cancelled all of my ECAs. 3) Is it true that international students have half the acceptance of a domestic student. | 1) I don't think international acceptance rates will be strongly affected by Covid. There's no good reason I can think of for colleges to accept fewer international students just because of Covid. 2) All students will struggle with this in some way. You will have to make the best of your situation. On the plus side, the pandemic gives an opportunity for students to show extra creativity where they otherwise wouldn't have had to. 3) Yes, unfortunately it is generally more competitive for international students to get into domestic schools. I believe it is a bit easier if you demonstrate you can pay tuition, but this isn't a big factor at the top universities. | |
10/11/2020 | Applications - Test Scores | Harvard declared itself as test optional but will not send scores affect my acceptance, I read multiple articles some say it will while others say it won't. I wanted to know what you thought about it. | While schools are test optional, I personally believe good test scores give you an advantage. If you don't send scores, they won't necessarily hold it against you, but there are just fewer points of data they can use to calibrate your performance vs other students. If you have some other notable achievement that convinces them you're in the top 1 percentile of students (like winning a competitive competition), then this can substitute for test scores. But if you don't, then it's easy to fall into the middle of the pack. | |
10/11/2020 | Applications - Recommendations | Is it bad to ask a freshman teacher to write your rec letter. I know this teacher who taught me freshman year and for a brief period sophomore year. However, I still talk to him a lot and feels like he knows me best and can write a great letter. The only problem is that he did not teach me junior or senior year. Do you think that will look bad? Thank you | The drawback to a freshman year teacher is that it'll have been over 2 years since he saw you in the classroom. People develop a lot from age 14 to age 17, and so his impression will be a bit outdated. Furthermore, colleges would wonder why it is you didn't develop a stronger relationship with any teacher since freshman year. Since you're a senior, you may not have much of a chance now to develop relationships with sophomore and junior year teachers, so if your freshman year teacher can best speak to your personality and interaction with other students, I would choose your freshman year teacher rather than a junior year teacher who doesn't know you as well. For younger students in this situatino, I would hedge a bit: continue your relationship with your freshman year teacher, but continue trying hard to develop relationships with sophomore and junior year teachers. Then when you apply, choose the best mix of teachers who can paint complementary portraits of you. | |
10/11/2020 | Coursework - APs | I am a rising sophomore in Canada. I have a few question regarding the AP and IB programs: 1. Are AP courses in Canada different than AP courses in the US(in terms of difficulty, course material, etc.)? 2. Do Ivy league universities prefer students who complete IB over students who take AP? I have heard that Ivy League's like to accept students who take more challenging coursework. Considering that IB is a more rigourous program than AP, would my chances of acceptance in an Ivy league school be higher if I took and IB? Thank you. | 1. AP _courses_ can vary a lot even within the same school, since it depends on how the school and teacher want to teach the curriculum. AP exams, however, are the same no matter where you are. 2. I've never heard of a big favoring of IB over AP curricula. I don't know the IB curriculum as well, but I've never heard of it being distinctly and significantly more difficult or challenging than AP - there are just different tradeoffs (e.g. in terms of breadth vs depth). I would choose whichever one you're more interested in or that's more convenient for you and not worry as much about it. | |
10/11/2020 | Applications - Chances | I have recently become a lot more interested in college admissions, and I have read your guides on getting better grades, SAT/ACT scores, and "How to get into Harvard." First off, I just want to say that these guides were awesome. They've given me a clear picture of what I need to do to get to where I want to be. Post college, I want to work on superconductor research. I think that developing a theory for superconductivity or simply creating new superconductors would be an excellent way for me to utilize my interests in physics and chemistry, and would also be a fulfilling line of work for me. To this end, I want to attend MIT. The issue here is that I am not confident that I can get into MIT. As a junior, I have a GPA of 3.65, and I have calculated that I can achieve a 3.8 GPA with the classes and time I have remaining at high school. I am also prepared to commit the time it would take to get a perfect score in either the SAT or ACT, or both if required. However, because I am a junior, I feel as though I might not be able to set up the "spike" that you have said would make me desirable for colleges. I enjoy physics and chemistry, and I have achieved either an A or B+ in AP Physics 1 and Chemistry. I've done a number of programs through an organization called "WCATY" (Wisconsin Center for Academically Talented Youth), which were predominantly 2 years of 3 week long courses at a nearby college, which allowed me to study physics in a much more strenuous environment than normal school. I've also got a small passion for history - I'm taking AP Euro this year, and was thinking about taking AP Psych in my senior year. Finally, I swim for my school, and am close to making varsity. My main question is then. - what else should I do? Like I said, I am assuming a best case scenario of a 3.8 GPA (my high school doesn't give more weight to AP classes), and a 1600 on the SAT, as well as a 36 on the ACT. Given those grades, and 6 AP classes in topics of my interest, how should I develop my spike? And finally, would it be worth pursuing any of the classes or clubs that I have laid out here? >My school has a couple of science competitions (Science Bowl and Physics Bowl, both of which extend beyond the school). >My school has a math team and a robotics team. >I am eligible for both National Honor Society and the Rho Kappa Soc. Stud. Honor Society. | Assuming that your academics are in place, I'd encourage you to explore the very interest you suggested: superconductor research. Can you get a research internship in this field somehow? If it's impossible, can you demonstrate your interest in this field another way (say, by making approachable videos teaching the content)? Schools like MIT and Caltech appreciate students who have a genuine interest in a field and do actual work on it. This can compensate for imperfect grades and test scores (though you can't fall too far). Lastly, I also want to stress that you don't _need_ to go to MIT to work on superconductor research. You should aim to get into your dream school, but don't be crushed if you don't get in - you can do cool research at many universities. | |
8/16/2020 | Coursework - APs | In my school, there is accelerated math 1 as a course and students who take that goes to accelerated math 2 in tenth grade. Then, AP Calculus AB in 11th grade. And, finally, AP Calculus BC in 12th grade. I completed ninth grade already and as a freshman, I took Accelerated math 1. But, the thing is, I really really struggled in that course (I ended up with B+) and I’m not sure if I want to take accelerated math 2 as a sophomore. Personally, I wish to major in the sciences, like biology or physics. So, I have two options here. 1) take accelerated math 2 as a sophomore although my grades won’t be really good. (I predict it be ‘B’. Maybe lower.) 2) take Algebra 2 trig as a sophomore. Go to precalculus as a junior. Then, go to AP Calculus AB as a senior. As you can see, going to algebra 2 trig won’t allow me to advance too far as a senior. Despite that, I get to have better grades and less stress in my high school life in regards to math subjects. Note: by going slower in my math curriculum, it won’t stop me from taking other courses that require prerequisites like AP Physics C or AP Chemistry. Personally, I think that GPA is more important than the courses I’m taking (although that’s still crucial). But, I’m not sure what to choose. What would you recommend? | I suggest challenging yourself to the point that you'll likely get an A but need to work for it. You don't want it so easy that you could have taken a more advanced course. But you don't want it so hard that you get a bad grade and stress yourself out a lot. It's also good that going slower in math won't block your prereqs for AP courses you want to take. In general, if your school has a policy for dropping courses by a specific time, you might try the harder course for a few weeks to see if you can handle the challenge. Then, if it's too much, you can drop the course or go down a level. | |
8/16/2020 | Thanks | I just wanted to say a huge THANK YOU! I am an international student who was studying for the ACT. I scored a 32 on my first test and was struggling to improve my score on the practice tests that I was doing. My parents don't have much money so I couldn't afford any tuition or online packages. I found Prepscholar just by searching up free ACT tips and tricks on the internet and it quickly became my main source of study. Using free practice tests and all your tips, I was able to improve my score in one test from a 32 to a perfect 36. I just wanted to thank you and encourage you to keep growing Prepscholar as I am sure it has helped so many students like myself. | Thank you for your thoughtful email. You put in all the hard work and you deserve your awesome score! This is going to be a huge help for your college applications. If you benefited from my advice, the only request I have for you is to pay it forward by helping younger students learn and achieve. Let people know about our resources. Then coach them on how best to improve like you did. | |
8/16/2020 | Applications - Course Requirements | Do i have to learn a second language in order to get into Harvard or any Ivy League school? | It's not absolutely required, but it is common. Here's what Harvard says: "There is no single academic path we expect all students to follow, but the strongest applicants take the most rigorous secondary school curricula available to them. An ideal four-year preparatory program includes four years of English, with extensive practice in writing; four years of math; four years of science: biology, chemistry, physics, and an advanced course in one of these subjects; three years of history, including American and European history; and four years of one foreign language." https://college.harvard.edu/resources/faq/are-there-secondary-school-course-requirements-admission I also think it's common in state graduation requirements to learn a second language, so your school likely is forcing you to do so anyway. | |
8/16/2020 | Extracurriculars - Academic Competitions | Today was the day that I received my AP scores, and I was pretty devastated to see a 3 on AP bio, the subject that I studied so hard on. That made me wonder if biology was the right subject for me to pursue, and I came to the conclusion that I either wanted to stay in sciences and do Chemistry and Physics as my spike or do machine learning and AI. As of now, I definitely have more experience in the science category, since I have started to learn Chemistry and have taken a year of Bio along with self studying for AP Bio exam. After reading your article and seeing your application to Harvard, it gave ideas on how to show my interest and passion in the science fields. I have emailed my teacher to ask if I can take AP Chemistry Sophomore year as an exception, but I was wondering if there were any places such as colleges or organizations that can help me excel in this area. I had no idea that there were science competitions specifically on Chemistry and Physics until I read your application, and I made it my goal to be able to compete in the National Chemistry Olympiad. However, as an incoming sophomore, the only thing I've done is study a bit on my own and with a tutor on basic chemistry. However, to be able to compete at the national level, I realized that I needed to know so many more than just basic chemistry. However, my school only offers up to AP Chemistry, and i was wondering if there were any other places that I can go to further my interest in Chem. I was hoping that I could join a professor at a college or watch a lab to see how it works in the real world, but I'm not sure how to do this. If you have any contacts or programs that I can do so that I can be fully prepared for the Olympiad by Junior year or possibly end of sophomore year, please let me know. I used to play a lot of hockey, and recently, I stopped because of the huge time commitment, and a lot of extra time has been offered to me. | From my memory, for the US National Chemistry Olympiad local and national exams, for the most part you need only up to AP Chemistry, with just a bit of organic chemistry. You can see the past exams here: https://www.acs.org/content/acs/en/education/students/highschool/olympiad/pastexams.html My best advice would be to take a look at the sample exams and study them like you would any other test. To get into the study camp, you'll basically need perfect scores on both tests. Once you figure out where you're weak, you need to drill those weaknesses. In terms of resources, you might consider joining online communities around the olympiad (there might be subreddits or forums), or hiring a tutor who specializes in prepping for the olympiad. | |
8/16/2020 | PrepScholar | I fit a similar profile to your "academic superstar". In 7th grade, I took the SAT through Duke University's TIP, and got a 1570 on it. Now, as a rising freshman, I practice test at consistently 1600. I'm incredibly interested in law and government as well: I took a practice LSAT for fun, did incredibly well, and registered for the August 2020 test. Now, I'm practice testing on the LSAT at anywhere from 176-180, in the top 0.2% of students eight years older than me. I'm working in the campaign for mayor of my city, *******. I have a burning passion for what I believe in and enjoy, and I know exactly what I want to do (and have known since the age of 9 years old.). Could you work with me on college admissions? | Thanks a lot for the note. You're clearly precocious and high-achieving, so you're in a good place. I personally don't have the bandwidth to offer college admissions services myself, but I personally created our admissions department here: http://admissions.prepscholar.com/ . You'll be able to find a college counselor who understands you and your background and will help coach you on your way to your future goals, whatever they are. As you mentioned, it's possible that you don't really need college application help. You have the luxury of not needing to worry about grades or test scores. This means you have plenty of free time to try a lot of things and figure out what you like, then get deep into your areas of interest. | |
8/16/2020 | Applications - Recommendations | I am currently unsure if my economics teacher is suitable as a teacher recommendation and was wondering if you would be able to give me some advice? I was not able to take economics at school because it was during my english periods, which are compulsory. Because I really wanted to take this subject, I decided to take it online without a teacher. However, every fortnight and frequently after school, I meet with my school economics teacher to ask and discuss any questions that I have encountered. He definitely knows my personality better than any other teacher at my school, and I think that out of all my teachers, he would be able to write the best recommendation. However, he has only taught me in Year 8, and since he is technically not my teacher now, would he be appropriate or should I ask another teacher who teaches me every week? | The important thing for a teacher recommendation is that the teacher knows you well - not just how you do academically, but your personality and how you engage with other students. The point of the recommendation is to touch on the soft skills that are so important to colleges - how you engage socially, how you overcome obstacles, what makes you special among the thousands of students they've taught. In your situation, it sounds like the teacher could comment on your curiosity and academic drive well, as well as your personal growth over the years. One weakness is that he might not be able to comment as much on your social interaction with other students. Also, if it's been years since he formally taught you, he might not be able to comment on more concrete aspects of your coursework, like how you did on projects and assignments. I'd hedge a bit if possible - continue developing your relationship with your Econ teacher, while also seeing if you could develop any relationships with your current teachers. Then, when you're applying, think about who your best recommenders would be and how two of them would complement each other. | |
8/16/2020 | Tests - SAT/ACT | i’m taking a online ACT prep course, by James Franklin and from this perspective he told me that it’s best to read ACT reading books like for example, The New York Times and etc. He said to try this 3+ months and a lot of his students tried it and it work, did u before also did this or did u just kept practicing on the ACT reading like who most people did. Please let me know thank you ! | I think reading well-written material is good for general development, and as a high school student reading The New Yorker and The Atlantic definitely helped develop my sense for good writing. However, hour for hour, I still think prepping specifically for the test is more effective. Reading is generally helpful, but it doesn't directly prep you for the test. Think of it this way - if you were training for a swim meet, weight lifting would probably help you, but not as much as using that same time on your swimming stroke. I also say this knowing most students have a finite amount of time and can't do full hours of test prep while also reading everything they want to read. | |
8/16/2020 | Applications - Essays | I’ve submitted my essay in many peer review websites online. Most have been encouraging and supportive of my essay but one individual struck it down as a “minority’s cliche”. My essay is about how I wear the Islamic headcovering of the hijab and how I used to be afraid of speaking about it but after several events, I now feel empowered and inspired by my religion to stand for justice. Is this too cliche and un engaging of a personal statement? | Online peer review sites have a few major problems: 1) you don't know who's reviewing it and whether they have any credibility, 2) they don't know anything about you or the rest of your application. Even worse, some toxic reviewers might deliberately want to make people feel bad about their applications. College admissions readers have read literal thousands of essays. There are only so many experiences people can possibly have in the first 17 years of their life, and they've seen it all. The important thing about your essay is that it needs to ring true to who you are. You shouldn't write from the mindset of trying to impress the admissions committee - this will come across as inauthentic. As long as you're writing about something you care about, and it demonstrates how you view the world, don't worry about it being a "cliche." Even if other people have had the same experience, how you interpreted it in your way is what matters. | |
8/16/2020 | Applications - Chances | I am a Nigerian and I am not based in the US. I am 15 years old. I will be done with high school by next year. I will love to make it into any of the Ivy league schools. Rest assured, I'm not the best in my class, but I can say, I'm actually smart. Like you said, being well rounded isn't the thing. I actually don't do much outside the classroom, because over here, there aren't much opportunities. My school doesn't do clubs, I write though (I've represented my school a number of times in external essay competitions). There aren't opportunities for clubs and I don't do sport. I don't actually play any instructions. I bake. I would love to study economics,because that's what I'm passionate about. My country's economy is really messed up, And I hope to come back home some day, and do something big here.I would also like to work in the World bank(actually, thats what i want.My country is an after thought).I would also like to change people's lives.Iuld take the SAT next year, and hope to get a really high score, with help from your platform as well as others. I also hope to do well in my final exam. What do you think are my chances of getting into any of the Ivy league schools? Is there anything I need to work on?Also, I would be applying for financial aid and scholarships because my parents can definitely not afford any of these schools. | Harvard and similar schools of course look for the top students in the world, and they're especially selective for international students (since they have so few slots for them). This means you'll ideally have strong academic performance and demonstrate deep achievement in your area of interest. Here's a guide on what it takes to increase your chances of getting into one of these schools. https://blog.prepscholar.com/how-to-get-into-harvard-and-the-ivy-league-by-a-harvard-alum If you have questions on your chances at getting into any particular school, my advice is to be ambitious and apply as though you have all the chances in the world. Whether someone like me tells you have a chance shouldn't really affect your thinking. If someone told you you have good chances, you need to avoid sitting on your laurels - you should keep pushing as hard as you can to exceed your goals. And if someone told you you're a reach, you shouldn't be demoralized - you should brush it off and keep pushing. So my opinion is not all that important - it's what you do internally that really matters at the end of the day. | |
8/16/2020 | Applications - Chances | I saw your article on Harvard and how they do their rankings. Because I have some international awards in dance I might qualify for a 1, but I am not certain about that. But my main question is, if I have near perfect Sat/ACT, grades, AP scholar with distinction , PSAT national merit, and French national exam gold medal. Do you think there is even a chance for me to get a 1 in the academic category? I am truly curious, as I definitely am going to focus on dance. I am definitely aware that to get a 1 in academics, it is not only perfect grades and scores. | (For background, check out my explanation of Harvard's rating system: https://blog.prepscholar.com/harvard-asian-admissions-lawsuit-application-strategy). A good way to think about a "1" as ranked by Harvard is that it signals a level of achievement that possibly only about 0.5% students applying that year will be able to achieve. And note - this is the top 0.5% of Harvard applicants, not of the entire nation. Only about 200 students per year get this score. As Harvard defines it, a 1 in an academic rating means: "1. Summa potential. Genuine scholar; near-perfect scores and grades (in most cases) combined with unusual creativity and possible evidence of original scholarship." Of 40,000 applicants a year, 8,000 had perfect GPAs, 625 had a perfect score on ACT; 361 had a perfect 2400 on SAT; 3,500 had perfect SAT math; 2,700 had perfect SAT verbal. So generally, perfect grades and AP scores are not enough to earn a 1. It requires some noteworthy original work or achievement that puts you within the top 0.5% of Harvard applicants, and possibly the top 0.05% within the nation | |
8/16/2020 | Extracurriculars - Research | I'm a rising junior, and and I'm planning to apply to some summer science enrichment programs between junior year and senior year (RSI, SSI, LaunchX, etc.). As an RSI alumni yourself, what tips/advice do you have for students involved with science, such as myself, for getting into these competitive programs? Is it a lot like the college admissions process? What differences are there? For RSI in particular, what test scores do you recommend to show? Outside of academics, what do you recommend students to do to optimize their application to these programs? | Applying to RSI is kind of like a mini version of applying to college. At a baseline, you should be academically strong, but these are just table stakes - there are thousands of students with near perfect test scores and GPAs. Beyond academics, you should show a special passion for your field of interest and ideally show prior achievement in it. This often means having already done meaningful research, or some other noteworthy extracurricular academic achievement (like national-level academic competitions). Your recommendations will also be important - RSI cares a lot about its community, and it doesn't want geniuses who are jerks. Your letters should show that you're an active participant in the classroom and get along well with your colleagues, if not be seen as a leader. | |
7/26/2020 | Tests - PSAT | I graduated from Harvard in 1993 (old man!) and one observation I made, at the time, was my perceived advantage students had in the admissions process that scored really high on the PSAT!! Nobody ever told me that in H.S. It was just, “this is simply practice for the SAT”. Think I scored 1100 and never thought about it again until I met people at Harvard who scored 1500 and then received letters from top schools! Was a real eye opener for me. I never want to push my kids too much, too early...they’ll be entering 7th and 9th grade, so I was entertaining the thought of your PSAT prep course. Was just wondering what YOUR thoughts were on the PSAT and college admissions. | Personally I think the PSAT/National Merit doesn't add much to the college admissions process - a high SAT score is far more important than a high PSAT score. A good thing about prepping for the PSAT is it preps the student for the SAT in a more approachable way, but for talented kids, it wouldn't be bad to just train on the SAT directly. | |
7/26/2020 | Tests - SAT/ACT | I struggle particularly with history passages in the SAT reading part, namely either the passage with two passages together or passages that are written by a historical person. It is very hard for me to understand these passages and thus I get a lot of the questions wrong, sometimes even up to half of the passage questions wrong. I have looked through almost all of PrepScholar's SAT Reading articles and have also used PrepScholar's classes and service but I just cannot improve. I know you might not be the most appropriate person to ask but I was just hoping you can give me some advice and tips. | It's hard to diagnose issues like this over email, but my biggest advice is to carefully review every single one of your mistakes. Do you really understand why you missed the question and how to avoid making that mistake in the future? If you have trouble with this, a tutor might be able to help. You can find a tutor with PrepScholar, or also through a lot of other sources. | |
7/26/2020 | Applications - Essays | I am a rising senior in the process of applying to colleges and I am requesting your assistance for the essay writing portion for my college list and potentially college apps as a whole. One of our family friends referred us to you, specifically you, as a great help during the process. | If you feel you could use extra help, you can get help from one of our superstar admissions counselors at http://admissions.prepscholar.com/. I created our admissions program myself, and they'll be the best people to help you with every aspect of your college applications. I suggest you sign up for a consultation to see if we're a good fit for you. | |
7/26/2020 | Coursework - Languages | I am a sophomore at Enloe High School. I have been taking Spanish since 7th grade. I just completed my 3rd year of Spanish in Freshman year. In your prepscholar blog, your colleagues mentioned that you should take Spanish all throughout high school. Does that apply to me as well since I'd finish 4 years of Spanish in 10th grade? | Typically 4 years of Spanish equates to something like AP level, so if you could take the AP exam in 10th grade and get a good score, then you've certainly done enough! | |
7/26/2020 | Coursework - APs | We have a question re my son (junior) and if he should take ap physics 1 (then ap physics c next yr) or if he should try for physics c this yr. his courses are: AP Calc BC AP Language AP AmHistory AP Physics (1 or C) Engineering Aerospace H (magnet) Adv Theater H (magnet) Theology3H He does XX competitions so that is 2.5-3hrs six days a week and XX which is nutso amount of hours all year. He has been able to pull off A’s but just not sure what he should do for physics. He wants to do C but not sure the work load and I worry because he takes so much on and wants to go to a top college for engineering. I am worried it is all too much n if maybe 1 is easier and will lighten the load? Or if C is necessary to be looked at for his goals. He likes the idea of taking math n science at a college his senior yr. | I generally suggest taking on whatever courseload is manageable to fit everything else in. Like any other single course, Physics 1 vs C is not a make or break thing. It's nice to have if it's a breeze; it's counterproductive if it sinks his other grades and makes him miserable. He'll ultimately have to see how feasible C is and whether he should drop down to 1. Consider checking out the material during the summer to see if he feels it's manageable or not. | |
7/26/2020 | Tests - SAT/ACT | I am starting from scratch (even though I got a GPA of 99.10 out of 100), and I am aiming to get a score of 1600 on the SAT. I have 840 hours for that and would like to know the best way to spend every single minute. I’m a high school graduate from Saudi Arabia who is planning to start applying to U.S universities by the 1st of August. I registered for the SAT test on December 5th. Now, for your information, as a Saudi student who has studied in the national section, my education was not concerned on preparing me for the SAT and I didn’t have the SAT in mind at all even after I graduated from high school. That said, I am still a student who scored excellent marks in high school subjects ( including maths and English ). Anyways, I am dedicating my full time for the SAT prep now. So my question is, while having in mind the time I am putting for this, do you think that your online prep program ( with tutoring 995$ ) can build me *from* * scratch * to where I want to be, or do you think that I should pair it with something else, or should I build myself first through Books and then come to you. And then also I don’t know if for example I should focus on maths for two months then reading and grammar for the other two months. | First thing I would say is take a breath - the stakes are high, yes, but you have time, and you don't have to feel like every millisecond is vital (every hour or day probably is though). I suggest focusing less on the materials you want to cram through and more on a dynamic approach - figure out where you are and iterate from there. First thing I would do is take a full-length practice test, as realistically as possible. What score do you get? If you get a 1200 you will need a different plan from if you get a 1550 the first try. If you don't know where you're scoring you have no business deciding what materials you're using. If you want a high score, these are the principles to use in your study: https://blog.prepscholar.com/how-to-get-a-perfect-sat-score-by-a-2400-sat-scorer - whether you use PrepScholar, books, or any other resource, these principles always hold true. Only after that would I worry about what resources to use and how much to study when. Give it a try. | |
7/26/2020 | Extracurriculars - Research | I currently live in Australia and was reading your 'My Successful Harvard Application' and was just wondering how could I possibly take up research positions at a science laboratory. I mean, wherever I search, I can only find internship opportunities for University students and not high school students. Could you give a suggestion about where and how I can land myself an internship as an year 10 high school student. | As with most things in life, you'll have to ask for what isn't handed to you. If you show interest and dedication and assure the lab staff that you're not going to waste their time, you'll be able to get a post somewhere. Be ambitious and aggressive and just ask, as best you can. The best way to get research opportunities at nearby colleges is to demonstrate 1) your interest and 2) your capability. For your interest, you should be interested specifically in what the lab is doing and have done your research on what they work on, then articulate why that interests you and what you would like to work on. For your capability, the point here is that most PI's don't want a student who wastes their time and isn't useful in pushing their research forward. You won't be as useful as a grad student, but demonstrate that you know the basics and are willing to learn really quickly, and that you'll stay committed for a long period of time (and not just for 3 months). And yeah, basically once you research a lab, you just need to write to the professor, or a grad student you're particularly interested in working with. Customize each reachout, don't just send 200 people the same email. | |
7/19/2020 | Extracurriculars | I can get into harvard law or science with music production as my spike, right? I mean if that is so then I’m definitely in luck. I know you’re not really familiar with music production like other passions ( science, writing, etc) but do you have any suggestions or tricks that can really boost my achievements in this craft (like opportunities, etc) and making it eye-catching to harvard? | Harvard doesn't require you to commit to a major like some other schools do, so yeah you can do something notable in any field and get in. In fact they want you to take some time and explore before committing. One suggestion for ideas is to think not just about how to be the top 0.1% music producer in the traditional sense and try to be the next Rick Rubin, but to combine it with other passions you have. For example: -music production x education (e.g. Youtube tutorials) -music production x community service (e.g. get kids into music production) -music production x computer programming (e.g. build cool music production tools) -music production x science (I don't have any ideas here but you can try 🙂 If you can be top 20% in music production and top 20% in field #2, then you're going to do quite well, and that doesn't require you to be a genius 0.1% music producer. | |
7/19/2020 | Tests - SAT/ACT | For a while now I've been studying for the SAT with the goal of taking it this fall. When I first started studying, I used Khan Academy, but after my mom and I read many of your college articles, we decided to give PrepScholar a try. PrepScholar has been helpful for me, I am very thankful for this program you have put together. Now that I'm in the 1500 range (1510 at the moment), I have the feeling that my habits and study strategies need to become even more efficient for me to work towards the perfect score, but I don't exactly know what to change moving forward. I feel a little frustrated at the moment because my family can't afford the full package (in any case I don't want individualized tutoring on every question), but I know I would greatly benefit from a one time meeting which I would use to get advice on improving my methodology, and to set my bearings straight for the last ninety points of my SAT journey. | My schedule is so booked that unfortunately I don't have the bandwidth to meet with the many students who ask for a meeting, so to be fair to everyone I can't take these meetings. But you've already made huge strides in your SAT score - improving from here on is doing more of the same, but to a higher degree of excellence. You need to be relentless about chasing down every single mistake and logging them so you never make the mistake again. Check out my Perfect SAT Score article on PrepScholar for more of this idea. I promise you that if you study intelligently, you will improve further from here. | |
7/19/2020 | Extracurriculars | I’m a grade 10 student currently living in Canada and i can say that I’m pretty good in academics. But as you said, I don’t want to be well-rounded and I want to have a spike. My passion was always music, I don’t know how to play instruments professionally, but i started learning music production and audio engineering when i was 12 and i can say that I’m really good at it. Became 3rd place in some local beatbattles, and 6th in a big world-wide competition with over 20,000 participants. But i haven’t achieved something that is world-changing. I don’t know what i can do to achieve something this great. Now I wanted to know, can this be my spike? If so how can i make this my spike? I also wanted to know does it matter if my spike is different than what I wanted to study at Harvard? For example spike: music / Harvard law | In brief, you don't have to do something world-changing in the traditional sense (eg starting a nonprofit). For example, Olympic athletes who win a silver medal arguably aren't changing the world necessarily, but they are extremely good at their craft and one of the world's best at what they do. That's enough for a spike. The point of a spike is that you demonstrate you have high potential to achieve in your area of passion. Colleges don't expect you to have found what your lifelong career passion is when you're 16. But you just need to demonstrate that once you do, you have the tenacity, creativity, and execution ability to do it. So I think your interest in music production definitely fits. Keep exploring that passion, keep developing your craft and pushing your boundaries of what you think is possible. You already seem like you're accomplished in it and passionate about it, and you can keep doing more in the next 18 months. No, your spike doesn't need to match what you want to study in college. | |
7/19/2020 | Extracurriculars | I am a student in the U.K. and I’ve always wanted to go to Harvard. After reading your article about your successful application and how to get into Ivy League schools, I’ve realised I do have a spike in science and research. However, when I think about how I could demonstrate that in an application, I’m not so sure it will be conveyed due to my lack of achievements. This is partly due to my high school focusing more on academic achievements or the ‘well rounded’ approach rather than specific specialisation. Seeing as I intend to apply for admission in September 2021, I’m not so sure how I can procure enough achievements to create a successful application, especially under the circumstances of the current global pandemic. I know I have a spike in science and research due to my contributions to class discussions which also led me to start a science magazine in my school. I also read many articles particularly related to innovation and document my opinions on them. | My best advice is to research what other people in your field of interest do to generate their spikes as ideas. You can read up on profiles in places like collegeconfidential (though this is a neurotic and sometimes toxic place, so don't spend too much time there). Once you have an idea, try to see how you can get the most progress on these within one year. | |
7/12/2020 | GPA | In seventh grade, I took Algebra 1, this course is a out of a 5.0 in my district. I ended it with an 89, a and do not know my weighted, or unweighted gpa. I wanted to know if I could improve this throughout the course of high schools and eighth grade geometry, or whether, I was stuck with it. I also wanted to know if this was a good start towards an Ivy League. It sounds a little far fetched, but I am only keeping my hopes high and my expectations low. | Generally colleges only look at your high school grades, so your 7th grade grades won't be factored into GPA. Furthermore, you don't need a perfect GPA to get in - they take into account many more factors. Here's a guide I wrote on how to get into elite schools: https://blog.prepscholar.com/how-to-get-into-harvard-and-the-ivy-league-by-a-harvard-alum | |
7/12/2020 | General | I am a rising 9th grader. In 2nd grade I had a dream of going to Harvard. I did not realize that getting into one of the best colleges in the world was hard and took time. Now, as I am getting closer to junior year, I realize that going to Harvard is harder than ever. I read your article on how to get into Harvard and I realized that what I have accomplished is mediocre. I was hoping you could help me out and give me tips and guides. I have accomplished things but not to the national level. I have many piano awards, but all are from regional to state. I did mathcounts but i failed terribly. I did Mock Trial where I received the Most effective attorney at state. I am also on the jv soccer team. I got a 1230 on PSAT and I am deeply worried about my future. My parents are also chinese, and they have high standards for me. If you could contact me and help me out that would be probably life changing! | Every person's path needs to be different: -figure out what you're interested in -think about what kind of impact you'd like to make on the world -set ambitious goals for yourself -figure out what you're missing in knowledge to achieve those goals. Get that knowledge -try and experiment. Fail a lot, but learn from your mistakes and keep iterating That's the general structure anyone should use to get through life, so it's the best advice I have for you. What exactly you do depends on your interests, and you need to determine that for yourself, not follow what some random person on the Internet tells you. | |
7/12/2020 | Extracurriculars | My passion in life is providing healthcare to low-income areas because I am a true believer in the fact that with resources and education, ignorance can be disbanded. As someone who has faced discrimination based upon ignorance in the past, this passion resonates with me to the bottom of my core. To create my "big spike" around my passion, I have dedicated my extracurricular activities to be centered around my passion. I am currently serving at a hospital as an intern, I hold a position in my schools ASB council as secretary, I created and am the president of debate club at my school, and I am taking college classes in foreign languages such as Arabic. At first, these extracurriculars can seem all over the place, however, they each go hand in hand to help me accomplish my passion. By serving as a hospital intern in a rigorous program, I am seeing the medical field first hand, by serving in my school ASB council and running debate club, I am aiding the next generation in developing critical thinking and disbanding ignorance in our community, and by taking foreign language classes, I am bridging the gap that lack of understanding different language creates, by allowing myself to be able to communicate with all types of people who need my help-seeking medical resources. However, after carefully reading your articles, I have begun to understand that this alone won't allow me to change my communities access to healthcare, and because of this realization, I have decided that I want to create a non-profit that works to help bridge the gap between low-income areas and their access to quality and formidable healthcare. The reason I am contacting you is that I need advice on how to make my non-profit a success and truly help people. | There is so much advice out there on how to build an organization and make it successful. My best advice to you is to learn as much as you can, apply what you learn quickly, then keep trying. The best book I would recommend for starting an organization is The Lean Startup. Beyond this, I would literally google "how to start a successful nonprofit," read through at least the top 10 links, come up with more questions, and keep Googling. Then at some point, actually try to implement what you've learned. You'll fail at some points but that's fine - just keep trying. |